UPSC Prelims 2023 GS 1 Question Paper with Answer Key, Explanation & Analysis [Download Now]

Guys! UPSC Prelims 2023 GS Paper 1 is just over now.

Sharing with you the question paper. 2024 Aspirants please go through these questions and tell us in the comments- how was the paper? which question did you find troubling? and let us solve the paper in the comment box.

Register and submit your doubts | 1-1 LIVE session with Sajal Sir and Zeeshan sir


Table of Content


Also, a LIVE analysis and 1-1 doubt session will be taken up by our team. We will answer and resolve all your doubts and come to know about the difficulty level.


Join Sajal sir and Zeeshan sir


UPSC Prelims 2023 Paper 1 is the GS Paper while UPSC Prelims 2023 Paper 2 is the CSAT exam.

Download the UPSC Prelims Question Paper 2023 PDF with answer keys & Explanation.

Download CSAT (Paper-II)


Answer Key

CivilsDailyIAS: UPSC CSE PRELIMS ANSWER KEY & EXPLANATION 2023

UPSC CSE PRELIMS 2023 (SET- B)
Ques NumberCorrect AnswerQues NumberCorrect AnswerQues NumberCorrect AnswerQues NumberCorrect Answer
1A26D51B76D
2B27A52B77A
3B28C53D78C
4D29B54C79A
5D30D55B80A
6C31B56B81C
7B32B57D82D
8A33B58D83A
9C34B59C84C
10D35D60C85B
11C36D61A86A
12C37C62B87D
13C38A63A88D
14B39D64D89C
15A40D65C90C
16D41D66C91D
17C42A67C92C
18B43A68D93D
19B44A69A94B
20C45B70D95B
21C46B71A96C
22C47C72B97A
23A48C73D98D
24B49B74A99C
25C50D75C100A

Solution with Explanation

SET: B

Q1. In which one of the following regions was Dhanyakataka, which flourished a prominent Buddhist center under the Mahasanghika , located?

(a) Andhra 

(b) Gandhara 

(c) Kalingo 

(d) Magadha

Answer: A

Dharanikota is a town near Amaravati in the Guntur district of Andhra Pradesh in India, It is the site of the ancient Dhanyakataka which was the capital of the Satavahana kingdom which ruled in the Deccan around the 1st to 3rd centuries A.D. 

Amaravati in Andhra Pradesh’s Guntur district is also known as Dhanyakataka or Dharanikota and was the site of a great Buddhist Stupa built in pre-Mauryan times, ruled by Satavahana kings.

Q2. With reference to ancient India, consider the following statements : 

1. The concept of Stupa is Buddhist in origin. 

2. Stupa was generally a repository of relics.

3. Stupa was a votive and commemorative structure in Buddhist tradition.

How many of the statements given above are correct?

(a) Only one 

(b) Only two 

(c) All three 

(d) None

Answer: B

Stupa, a Buddhist commemorative monument usually housing sacred relics associated with the Buddha or other saintly persons.

Religious buildings in the form of the Buddhist stupa, a dome shaped monument, started to be used in India as commemorative monuments associated with storing sacred relics of the Buddha.

Archaeologists in India have observed that a number of early Buddhist stupas or burials are found in the vicinity of much older, pre-historic burials, including megalithic sites.[9][10] This includes site associated with the Indus Valley civilization where broken Indus-era pottery was incorporated into later Buddhist burials.

Q3. With reference to ancient South India, Korkai, Poompuhar and Muchiri were well-known as-

(a) Capital cities

(b) Ports

(c) Centres of iron-and-steel making

(d) Shrines of Jain Tirthankaras

Answer: (b)

Poompuhar is a town in the Mayiladuthurai district in the southern Indian state of Tamil Nadu. It was once a flourishing ancient port city known as Kaveri Poompattinam and Kaveripattinam (not to be confused with modern Kaveripattinam), which for a while served as the capital of the early Chola kings in Tamilakam. 

Korkai was the capital, principal center of trade and important port of the Early Pandyan Kingdom. At that time, it was located on the banks of the Tamiraparani River and at the sea coast, forming a natural harbour. Due to excessive sedimentation, the sea has receded about 6 km in the past 2000 years, leaving Korkai well inland today.

Muziris found mention in the Periplus of the Erythraean Sea, the bardic Tamil poems and a number of classical sources.It was the major ancient port city of Cheras. Core of the city is situated in the present day Kodungallur, North Paravoor area. 

https://www.civilsdaily.com/type/places-in-news/page/3/

Q4. Which one of the following explains the practice of “Vattakirutal’ as mentioned in Sangam poems?

(a) Kings employing women bodyguards.

(b) Learned persona assembling in royal courts to discuss religious and philosophical matters

(c) Young girls keeping watch over agricultural fields and driving away birds and animals

(d) A king defeated in a battle committing ritual suicide by starving himself to death

Answer: D

Vatakkiruttal, also Vadakiruthal and vadakiruttal, was a Tamil ritual of fasting till death. It was especially widespread during the Sangam age. The Tamil kings, in order to save their honour, and prestige, were prepared to meet their death facing North (‘Vatakkiruttal’) and never would they turn their back in battle. It was a Tamil martial art. This was either done alone, or as a group with the supporters of the captured king.

Q5.  Consider the following dynasties:

1. Hoysala 

2. Gahadavala 

3. Kakatiya 

4. Yadava

How many of the above dynasties established their kingdoms in the early eighth century AD? 

(a) Only one 

(b) Only two 

(c) Only three 

(d) None

Answer: D

The Hoysala Empire was a Kannadiga power originating from the Indian subcontinent that ruled most of what is now Karnataka between the 10th and the 14th centuries. The capital of the Hoysalas was initially located at Belur, but was later moved to Halebidu.

The Kakatiya dynasty was a Telugu dynasty that ruled most of eastern Deccan region in present-day India between the 12th and 14th centuries. Their territory comprised much of the present day Telangana and Andhra Pradesh, and parts of eastern Karnataka, northern Tamil Nadu, and southern Odisha. Their capital was Orugallu, now known as Warangal.

The Seuna, Sevuna, or Yadavas of Devagiri was a Medieval Indiandynasty, which at its peak ruled a kingdom stretching from the Narmada river in the north to the Tungabhadra river in the south, in the western part of the Deccan region. The Yadavas initially ruled as feudatories of the Western Chalukyas. Around the middle of the 12th century, as the Chalukya power waned, the Yadava king Bhillama V declared independence. The Yadava kingdom reached its peak under Simhana II, and flourished until the early 14th century, when it was annexed by the Khalji dynasty of the Delhi Sultanate in 1308 CE.

The Gahadavala dynasty also Gahadavalas of Kannauj was a Rajput dynasty that ruled parts of the present-day Indian states of Uttar Pradesh and Bihar, during 11th and 12th centuries. Their capital was located at Banaras (now Varanasi) in the Gangetic plains, and for a brief period, they also controlled Kannauj.

Q6. With reference to ancient Indian History, consider the following pairs 

Literary work : Author 

1. Devi Chandragupta : Bilhana 

2. Hammira-Mahakavya : Nayachandra Suri 

3. Melinda panha : Nagarjuna 

4. Nitivakyamrita : Somadeva Suri

How many of the above pairs are correctly matched? 

(a) Only one 

(b) Only two 

(c) Only three 

(d) All four

Answer: C

Devi-Chandraguptam or Devi-Chandragupta is an Indian Sanskrit-language political drama attributed to Vishakhadeva, who is generally identified with Vishakhadatta. 

Hammira Mahakavya is a 15th-century Indian Sanskrit epic poem written by the Jain scholar Nayachandra Suri. 

The Milindapanho is a Buddhist text from sometime between 100 BC and 200 AD written by Nagasena. It purports to record a dialogue between the Indian Buddhist sage Nagasena, and the Indo-Greek king Menander of Bactria, who reigned in the 2nd century BC.

Nītivākyāmṛta —The Jaina writer Somadeva of tenth century A.D. writes a treatise on nīti is called Nītivākyāmṛta. He quotes fragments of the metrical works on polity attributed to Atri, Aṅgīras, Kauśika, Garga, Devala, Nārada, Parāśara, Bhāguri, Bhāradvāja, Bhṛgu, Bṛhaspati, Śukra, Vyāsa etc.

Q7. ‘Souls are not only the property of animal and plant life, but also of rocks, running water and many other natural objects not looked on as living by other religious sects’. The above statement reflects one of the core beliefs of which one of the following religious sects of ancient India? 

(a) Buddhism 

(b) Jainism 

(c) Shaivism 

(d) Vaishnavism

Answer: B

The core belief that souls are not only the property of animal and plant life but also of rocks, running water, and many other natural objects not looked upon as living things is associated with Jainism.

Jainism is an ancient Indian religion that places a strong emphasis on non-violence, compassion, and the principle of ahimsa (non-harming). One of the fundamental teachings in Jainism is the belief in the existence of souls (jivas) in all living beings, including humans, animals, and plants. However, Jain philosophy goes beyond this and extends the concept of souls to non-living entities as well, including rocks, water, and other natural objects. According to Jainism, all entities possess a soul, regardless of whether they are traditionally considered living or not.

This belief in the inherent presence of souls in various natural objects is a unique aspect of Jainism’s worldview and distinguishes it from other major Indian religions such as Buddhism, Vaishnavism, and Shaivism.

Q8. Who among the following rulers of Vijayanagara Empire constructed a large dam across Tungabhadra River and a canal-cum-aqueduct several kilometers long from the river to the capital city? 

(a) Devaraya I 

(b) Mallikarjuna 

(c) Vira Vijaya 

(d) Virupaksha

Answer: A

The large dam across the Tungabhadra River and the canal aqueduct several kilometers away from the capital city in the Vijayanagara Empire were constructed during the reign of King Krishna Deva Raya. Krishna Deva Raya was one of the most famous rulers of the Vijayanagara Empire, and he undertook numerous infrastructure projects to improve the irrigation and agricultural capabilities of his kingdom.

The credit for making the capital city of the Vijayanagara Empire one of the biggest cities in the 15th century goes to Deva Raya I. He realized that the scarcity of water supply, both for drinking and for irrigation, was restricting the growth of the royal capital. In c.1410 he had a barrage constructed across the Tungabhadra river and commissioned a 24 km long aqueduct from the Tungabhadra river to the capital.

Q9. Who among the following rulers of medieval Gujarat surrendered Diu to the Portuguese? 

(a) Ahmad Shah 

(b) Mahmud Begarha 

(c) Bahadur Shah 

(d) Muhammad Shah

Answer: C

The ruler of Gujarat who surrendered Diu to the Portuguese was Sultan Bahadur Shah. In 1535, after a long siege, Sultan Bahadur Shah was compelled to surrender the island of Diu to the Portuguese led by Nuno da Cunha. This event marked the beginning of Portuguese control and influence in the region, particularly in the maritime trade routes of the Indian Ocean.

1538 Siege of Diu resulted in the permanent occupation of Diu by Portuguese which lasted till 1961.

Q10. By which one of the following Acts was the Governor General of Bengal designated as the Governor General of India?

(a) The Regulating Act 

(b) The Pitt’s India Act 

(c) The Charter Act of 1793 

(d) The Charter Act of 1833

Answer: D

The office of the Governor General of Bengal was designated as the Governor General of India through the Government of India Act of 1833. This act, also known as the Charter Act of 1833, was passed by the British Parliament and granted expanded powers to the Governor General of Bengal, transforming the position into the Governor General of India. Lord William Bentinck was the first Governor General of India under this new designation, serving from 1833 to 1835.

Q11. In essence, what does ‘Due Process of Law’ mean? 

(a) The principle of natural justice 

(b) The procedure established by law 

(c) Fair application of law 

(d) Equality before law

Answer: C

Due process refers to just, rational, fair, and fair treatment under the regular judicial process. For instance, the accused must be given the chance to present their own defence before being sentenced.

The “Procedure Established by Law” means that a law duly enacted by the legislature or the concerned body is valid only if the correct procedure has been followed to the letter.

“Due Process of Law” is a doctrine that not only checks if there is a law to deprive the life and personal liberty of a person but also ensures that the law is made fair and just.

Q12. Consider the following statements 

Statement-I : In India, prisons are managed by State Governments with their own rules and regulations for the day-to-day administration of prisons. 

Statement-II : In India, prisons are governed by the Prisons Act, 1894 which expressly kept the subject of prisons in the control of Provincial Governments.

Which one of the following is correct in respect of the above statements? 

(a) Both Statement-I and Statement-II are correct and Statement-II is the correct explanation for Statement-I 

(b) Both Statement-I and Statement-II are correct and Statement-I1 is not the correct explanation for Statement-I 

(c) Statement-I is correct but Statement-II is incorrect 

(d) Statement-I is incorrect but Statement-II is correct

Answer: C

‘Prisons’ is a State subject under the State List of the Seventh Schedule to the Constitution of India. So, the management and administration of Prisons fall exclusively in the domain of the State Governments.

As per the provisions of Constitution of India, ‘prisons’/ ‘persons detained therein’ is a ‘State’ subject. The responsibility of prison management and prisoners administration solely vests with State Governments who alone are competent to make appropriate legislative provisions in this regard.

The 1894 Act dealt with provisions for accommodation, food, clothing, bedding segregation, and the discipline of prisoners, including solitary confinement. It also laid down provisions for the prisoners’ employment, health, and visits.

Q13. Which one of the following statements best reflects the Chief purpose of the `Constitution’ of a country? 

(a) It determines the objective for the making of necessary laws. 

(b) It enables the creation of political offices and a government. 

(c) It defines and limits the powers of government. 

(d) It secures social justice, social equality and social security.

Answer: C

The primary purpose of a constitution is to establish the framework for governance and define the structure of the government. It outlines the powers and functions of various branches of government, such as the executive, legislative, and judiciary. 

While the other options may be important aspects or goals associated with a constitution, such as the creation of political offices and a government (B) or securing social justice, social equality, and social security (D), they do not capture the fundamental role of a constitution in defining and limiting the powers of the government.

Q14. In India, which one of the following Constitutional Amendments was widely believed to be enacted to overcome the judicial interpretations of Fundamental Rights?               

(a) 1st Amendment                 

(b) 42nd Amendment                

(c) 44th Amendment                 

(d) 86th Amendment               

Answer: B

The constitutional amendment in India that was widely believed to be enacted to overcome the judicial interpretations of the fundamental rights is the 42nd Amendment Act of 1976.

The Parliament was given unrestrained power to amend any parts of the Constitution, without judicial review. This essentially invalidated the Supreme Court’s ruling in Kesavananda Bharati v. State of Kerala in 1973. The amendment to article 368, prevented any constitutional amendment from being “called in question in any Court on any ground”. It also declared that there would be no limitation whatever on the constituent power of Parliament to amend the Constitution.

The 42nd Amendment Act is often referred to as the “Mini Constitution” or the “Constitution of Indira.” It was enacted during the period of emergency rule in India and introduced several significant changes to the Constitution. One of the main objectives of the amendment was to curtail judicial interpretations that expanded the scope and enforcement of fundamental rights.

The amendment sought to restrict the power of the judiciary and grant more authority to the Parliament in matters related to constitutional interpretation. It added certain provisions like Article 31-C, which aimed to limit the judicial review of laws affecting socio-economic rights. The amendment also introduced changes to the powers of the President, the Prime Minister, and the state governments.

The 42nd Amendment Act was met with criticism for its perceived erosion of judicial independence and dilution of fundamental rights. Subsequently, many provisions of the amendment were revised or repealed by subsequent amendments, restoring some of the original balance between the judiciary and the executive.

Q15. Consider the following organizations/ bodies in India :

 1.  The National Commission  for Backward Classes

 2.  The National Human Rights Commission                   

 3.  The National Law Commission  

 4.  The National Consumer Disputes Redressal Commission          

How many of the above are constitutional bodies?           

 (a) Only one

 (b) Only two                     

 (c) Only three                   

 (d) All four

Answer: A

The National Commission for Backward Classes is an Indian constitutional body under the jurisdiction of Ministry of Social Justice and Empowerment, Government of India established through Constitution Act, 2018 (also called, 102nd Amendment Act, 2018) this amendment act in the constitution to make it a constitutional body under Article 338B of the Indian Constitution. It was constituted pursuant to the provisions of the National Commission for Backward Classes Act, 1993

The National Human Rights Commission of India is a statutory body constituted on 12 October 1993 under the Protection of Human Rights Ordinance of 28 September 1993. It was given a statutory basis by the Protection of Human Rights Act, 1993.

Law Commission of India is a non-statutory body and is constituted by a notification of the Government of India, Ministry of Law & Justice, Department of Legal Affairs with a definite terms of reference to carry out research in the field of law and the Commission makes recommendations to the Government 

The National Consumer Disputes Redressal Commission of India is a quasi-judicial commission in India which was set up in 1988 under the Consumer Protection Act, 1986. Its head office is in New Delhi. The commission is headed by a sitting or retired judge of the Supreme Court of India.

Q16. Consider the following statements : 

1. If the election of the President of India is declared void by the Supreme Court of India, all acts done by him/her in the performance of duties of his/her office of President before the date of decision become invalid. 

2. Elections for the post of the President of India can be postponed on the ground that some Legislative Assemblies have been dissolved and elections are yet to take place. 

3. When a Bill is presented to the President of India, the Constitution prescribes time limits within which he/she has to declare his/her assent. 

How many of the above statements are correct? 

(a) Only one 

(b) Only two 

(c) All three 

(d) None

Answer: (d) 

If the election of the President of India is declared void by the Supreme Court of India, all acts done by him/her in the performance of duties of his/her office of President before the date of the decision become invalid: This statement is incorrect. If the election of the President of India is declared void by the Supreme Court, it does not render all acts done by the President in the performance of their duties before the date of the decision as invalid. The President’s actions and decisions taken during their tenure remain valid unless specifically declared otherwise.

Elections for the post of the President of India can be postponed on the ground that some Legislative Assemblies have been dissolved and elections are yet to take place: This statement is incorrect. The elections for the post of the President of India cannot be postponed solely on the ground that some Legislative Assemblies have been dissolved and elections are yet to take place. The President’s election follows a specific schedule and process outlined in the Constitution.

When a Bill is presented to the President of India, the Constitution prescribes time limits within which he/she has to declare his/her assent: This statement is incorrect. The Constitution of India does not prescribe specific time limits within which the President has to declare their assent when a bill is presented to them. The President has the power to return a bill for reconsideration, withhold their assent, or give their assent based on their discretion.

Q17. With reference to Finance Bill and Money Bill in the Indian Parliament Money the following statements :  

1. When the Lok Sabha transmits the Finance Bill to the Rajya Sabha, it can amend or reject the Bill. 

2. When the Lok Sabha transmits Money Bill to the Rajya Sabha, it cannot amend or reject the Bill, it can only make recommendations. 

3. In the case of disagreement between the Lok Sabha and the Rajya Sabha, there is no joint sitting for Money Bill, but a joint sitting becomes necessary for the Finance Bill.

How many of the above statements are correct?

(a) Only one 

(b) Only two 

(c) All three 

(d) None

Answer: (c) All three

When the Lok Sabha transmits the Finance Bill to the Rajya Sabha, it can amend or reject the Bill: This statement is correct. The Finance Bill, being a Money Bill, can be amended or rejected by the Rajya Sabha. However, any amendments made by the Rajya Sabha must be accepted or rejected by the Lok Sabha. The Lok Sabha has the final say on the Finance Bill.

When the Lok Sabha transmits the Money Bill to the Rajya Sabha, it cannot amend or reject the Bill, it can only make recommendations: This statement is correct. As per Article 109 of the Indian Constitution, the Rajya Sabha does not have the power to amend or reject a Money Bill. It can only make recommendations for amendments, which are not binding on the Lok Sabha. The Lok Sabha has the final authority to accept or reject these recommendations.

In the case of disagreement between the Lok Sabha and the Rajya Sabha, there is no joint sitting for the Money Bill, but a joint sitting becomes necessary for the Finance Bill: This statement is correct. In the case of a disagreement between the two houses on a Money Bill, there is no provision for a joint sitting to resolve the matter. The decision of the Lok Sabha is considered final. However, if there is a disagreement on a non-Money Bill, including the Finance Bill, a joint sitting of both Houses of Parliament can be called to resolve the disagreement.

Q.18) Consider the following statements:

Once the Central Government notifies an area as a ‘Community Reserve’ 

1. the Chief Wildlife Warden of the State becomes the governing authority of such forest 

2. hunting is not allowed in such area 

3. people of such area are allowed to collect non-timber forest produce 

4. People of such area are allowed traditional agricultural practices 

How many of the above statements are correct?

(a) Only one 

(b) Only two 

(c) Only three 

(d) All four

Answer: (b)

  1. Once the Central Government notifies an area as a ‘Community Reserve’, the Chief Wildlife Warden of the State becomes the governing authority of such forest: This statement is correct. When an area is notified as a ‘Community Reserve’, the Chief Wildlife Warden of the State or a designated authority becomes responsible for the management and governance of the reserve.
  2. Hunting is not allowed in such area: This statement is correct. In a Community Reserve, hunting is strictly prohibited to protect and conserve wildlife.
  3. People of such area are allowed to collect non-timber forest produce: This statement is incorrect. In a Community Reserve, the collection of non-timber forest produce by the local communities may be regulated or restricted to ensure sustainable use and conservation of resources. It is subject to the specific rules and regulations governing the Community Reserve.
  4. People of such area are allowed traditional agricultural practices: This statement is incorrect. Traditional agricultural practices are generally not allowed within a Community Reserve. The primary focus of a Community Reserve is to conserve biodiversity and natural habitats, which may require restricting agricultural activities to protect the ecological integrity of the area.

Q19.  With reference to ‘Scheduled Areas’ in India, consider the following statements:

1. Within a State, the notification of an area as Scheduled Area takes place through an Order of the President.

2. The largest administrative unit forming the Scheduled Area is the District and the lowest is the cluster of villages in the Block. 

3. The Chief Ministers of concerned States are required to submit annual reports to the Union Home Ministry on the administration of Scheduled Areas in the States.

How many of the above statements are correct?

(a) Only one

(b) Only two

(c) All three

(d) None

Answer: (b) Only two

  1. Within a State, the notification of an area as Scheduled Area takes place through an Order of the President: This statement is correct. The notification of an area as a Scheduled Area within a State is done through a Presidential Order. The President has the authority to declare certain areas as Scheduled Areas based on the criteria specified in the Constitution.
  2. The largest administrative unit forming the Scheduled Area is the District and the lowest is the cluster of villages in the Block: This statement is correct. In a Scheduled Area, the administrative units are organized from the district level to the cluster of villages in the Block. The District is the largest administrative unit, and within the District, there are smaller administrative units such as Blocks and Clusters of villages.
  3. The Chief Ministers of concerned States are required to submit annual reports to the Union Home Ministry on the administration of Scheduled Areas in the States: This statement is incorrect. In reality, it is the Governor of the concerned State who is responsible for submitting annual reports to the President of India, not the Chief Ministers. The Governor acts as the representative of the President in the State and is responsible for reporting on the administration and development activities in the Scheduled Areas to the President through the Union Home Ministry.

Q20. Consider the following statements :

Statement-I : The Supreme Court of India has held in some judgments that the reservation policies made under Article 16(4) of the Constitution of India would be limited by Article 335 for maintenance of efficiency of administration.

Statement-II : Article 335 of the Constitution of India defines the term ‘efficiency of administration’.

Which one of the following is correct in respect of the above statements? 

(a) Both Statement-I and Statement-II are correct and Statement-II is the correct explanation for Statement-I 

(b) Both Statement-I and Statement-II are correct and Statement-II is not the correct explanation for Statement-I 

(c) Statement-I is correct but Statement-II is incorrect

(d) Statement-I is incorrect but Statement-II is correct

Answer: (c)

Statement-I: The Supreme Court of India has held in some judgments that the reservation policies made under Article 16(4) of the Constitution of India would be limited by Article 335 for the maintenance of efficiency of administration: This statement is correct. The Supreme Court has indeed held in certain judgments that while reservation policies can be implemented under Article 16(4) to provide opportunities to backward classes, they should also be balanced with the consideration of maintaining the efficiency of administration, as stated in Article 335.

Statement-II: Article 335 of the Constitution of India defines the term ‘efficiency of administration’: This statement is incorrect. Article 335 does not provide a specific definition of ‘efficiency of administration’. Instead, it emphasizes the need to balance the claims of the Scheduled Castes and Scheduled Tribes (SC/ST) with the maintenance of efficiency in administration. Article 335 states that the claims of the SC/ST candidates should be taken into consideration in appointments and posts under the State, but without compromising the efficiency of administration.

Q21.  Consider the following statements: 

Statement-I: India, despite having uranium deposits, depends on coal for most of its electricity production.

Statement-II: Uranium, enriched to the extent of at least 60%, is required for the production of electricity.

Which one of the following is correct in respect of the above statements? 

(a) Both Statement-I and Statement-II are correct and Statement-II is the correct explanation for Statement-I 

(b) Both Statement-I and Statement-II are correct and Statement-II is not the correct explanation for Statement-I 

(c) Statement-I is correct but Statement-II is incorrect

(d) Statement-I is incorrect but Statement-II is correct

Answer: (c) Statement-I is correct but Statement-II is incorrect.

Statement-I: India, despite having uranium deposits, depends on coal for most of its electricity production: This statement is correct. India does have significant uranium deposits, but its reliance on coal for electricity production is higher compared to uranium. Coal continues to be the primary source of energy for electricity generation in India due to its abundant availability and existing infrastructure.

Statement-II: Uranium, enriched to the extent of at least 60%, is required for the production of electricity: This statement is incorrect. While uranium is used as a fuel for nuclear power generation, the level of enrichment required for electricity production is not fixed at 60%. The enrichment level can vary depending on the type of reactor and its design. Typically, for light-water reactors, the enrichment level is around 3-5% uranium-235, not 60%.

Q22.  Consider the following statements:

Statement-I: Marsupials are not naturally found in India.

Statement-II: Marsupials can thrive only in montane grasslands with no predators.

Which one of the following is correct in respect of the above statements? 

(a) Both Statement-I and Statement-II are correct and Statement-II is the correct explanation for Statement-I 

(b) Both Statement-I and Statement-II are correct and Statement-II is not the correct explanation for Statement-I 

(c) Statement-I is correct but Statement-II is incorrect

(d) Statement-I is incorrect but Statement-II is correct

Answer: (c) Statement-I is correct but Statement-II is incorrect.

Statement-I: Marsupials are not naturally found in India.

This statement is correct. Marsupials, a group of mammals characterized by carrying their young in a pouch, are not naturally found in India. They are primarily found in Australia and nearby regions.

Statement-II: Marsupials can thrive only in montane grasslands with no predators.

This statement is incorrect. While some marsupials, such as kangaroos and wallabies, are known to inhabit grassland habitats, they are not limited to montane grasslands. Marsupials can be found in a variety of habitats, including forests, woodlands, and even deserts. Additionally, their survival is not limited to areas without predators, as marsupials have evolved various adaptations to coexist with predators.

Q23.  ‘Invasive Species Specialist Group’ (that develops Global Invasive Species Database) belongs to which one of the following organizations?

(a) The International Union for Conservation of Nature

(b) The United Nations Environment Programme

(c) The United Commission for Environment and Nations World Development

(d) The World Wide Fund for Nature

Answer: (a) The International Union for Conservation of Nature (IUCN).

The ‘Invasive Species Specialist Group’ (ISSG), which develops the Global Invasive Species Database, belongs to the International Union for Conservation of Nature (IUCN). The IUCN is a global organization dedicated to conserving nature and promoting sustainable development. It provides a platform for governments, NGOs, scientists, and experts to collaborate on conservation efforts and develop strategies to address various environmental challenges.

The ISSG, as a specialist group within the IUCN, focuses specifically on invasive species and their impacts on biodiversity and ecosystems. It works to raise awareness, conduct research, and develop tools and resources related to invasive species management. The Global Invasive Species Database is one of its notable initiatives, providing valuable information on invasive species worldwide.

Therefore, the ‘Invasive Species Specialist Group’ belongs to the International Union for Conservation of Nature (IUCN).

Q24.  Consider the following fauna:

1. Lion-tailed Macaque

2. Malabar Civet

3. Sambar Deer

How many of the above are generally nocturnal or most active after sunset?

(a) Only one

(b) Only two

(c) All three

(d) None

Answer: (b)

Among the given fauna, two of them are generally nocturnal or most active after sunset:

  1. Lion-tailed Macaque: Lion-tailed Macaques (Macaca silenus) are diurnal primates, which means they are active during the day and rest at night. They are not generally nocturnal or most active after sunset.
  2. Malabar Civet: The Malabar Civet (Viverra civettina) is a nocturnal mammal found in the Western Ghats of India. It is primarily active during the night, making it nocturnal.
  3. Sambar Deer: The Sambar Deer (Rusa unicolor) is a large deer species found in various habitats across India. They are generally crepuscular, which means they are most active during dawn and dusk. While they may be active during the night, they are not strictly nocturnal.

Based on the explanations above, the correct answer is (b) Only two.

Q25. Which of the following organisms perform a waggle dance for others of their kin to indicate the direction and the distance to a source of their food?  

(a) Butterflies 

(b) Dragonflies 

(c) Honeybees 

(d) Wasps 

Answer: (c) Honeybees.

Honeybees are known to perform a waggle dance as a means of communication within their colony. This dance is performed by worker bees to indicate the direction and distance to a source of food, such as nectar or pollen. The waggle dance involves the bee moving in a figure-eight pattern while waggling its abdomen and producing buzzing sounds. By observing the direction and intensity of the waggle dance, other worker bees can determine the location of the food source and navigate to it. This behavior is a remarkable example of how honeybees communicate and share information within their colony.

https://www.science.org/doi/10.1126/science.ade1702

Q.26)   Consider the following statements:

1. Some mushrooms have medicinal properties.

2. Some mushrooms have psycho- active properties.

3. Some mushrooms have insecticidal properties.

4. Some mushrooms have biolumi- nescent properties.

How many of the above statements are correct?

(a) Only one

(b) Only two

(c) Only three

(d) All four

Answer: (d) All four.

  1. Some mushrooms have medicinal properties: Certain mushrooms possess bioactive compounds that exhibit medicinal properties. For example, species like Reishi (Ganoderma lucidum) and Turkey Tail (Trametes versicolor) have been traditionally used for their potential health benefits.
  2. Some mushrooms have psychoactive properties: Certain mushrooms contain psychoactive compounds such as psilocybin and psilocin, which can induce hallucinogenic effects. These mushrooms are often referred to as “magic mushrooms” or “psilocybin mushrooms.”
  3. Some mushrooms have insecticidal properties: Certain species of mushrooms contain natural insecticidal compounds that can be used as a natural alternative to synthetic insecticides. For example, the entomopathogenic fungus Beauveria bassiana is used as a biocontrol agent against insect pests.
  4. Some mushrooms have bioluminescent properties: There are certain species of mushrooms, such as the Jack-O’-Lantern mushroom (Omphalotus olearius), that have bioluminescent properties. They emit a faint greenish glow in the dark due to the presence of luciferin-luciferase reaction.

Q.27)  Consider the following statements regarding the Indian squirrels :

1. They build nests by making burrows in the ground.

2. They store their food materials like nuts and seeds in the ground.

3. They are omnivorous.

How many of the above statements are correct?

(a) Only one

(b) Only two

(c) All three

(d) None

Answer: (a)

Statement 1 is incorrect: Indian squirrels, such as the Indian palm squirrel (Funambulus spp.), do not build nests by making burrows in the ground. They usually build nests called dreys in tree branches or tree hollows.

Statement 2 is incorrect: Squirrels do not typically store their food materials like nuts and seeds in the ground. They often create small caches in various locations, such as tree hollows or crevices, to store their food for later consumption.

Statement 3 is correct: Indian squirrels, including the Indian palm squirrel, are primarily herbivorous and feed on a diet consisting mainly of plant material such as fruits, nuts, seeds, and tree bark.

Therefore, the correct answer is (c) All three.

Q.28)  Consider the following statements: 

1. Some microorganisms can grow in environments with temperature above the boiling point of water. 

2. Some microorganisms can grow in environments with temperature below the freezing point of water.

3. Some microorganisms can grow in a highly acidic environment with a pH below 3.

How many of the above statements are correct?

(a) Only one

(b) Only two

(c) All three

(d) None

Answer: (c) All three.

  1. Some microorganisms can grow in environments with temperatures above the boiling point of water. These organisms are known as thermophiles and can withstand and thrive in extreme heat, such as hot springs or deep-sea hydrothermal vents.
  2. Some microorganisms can grow in environments with temperatures below the freezing point of water. These organisms are known as psychrophiles or cryophiles and can survive and reproduce in extremely cold conditions, such as glaciers or Arctic regions.
  3. Some microorganisms can grow in highly acidic environments with a pH below 3. These organisms are known as acidophiles and have adapted to acidic conditions, such as acid mine drainage or volcanic areas.

Q.29)  Which one of the following makes a tool with a stick to scrape insects from a hole in a tree or a log of wood?

(a) Fishing cat

(b) Orangutan

(c) Otter

(d) Sloth bear

Answer: (b) Orangutan

Orangutans are known to use tools in their natural behavior, including making tools with sticks to scrape insects from holes in trees or logs of wood. They exhibit remarkable problem-solving abilities and tool use in their foraging activities. This behavior is observed in both wild and captive orangutans and is a demonstration of their intelligence and adaptability.

Q.30)  Consider the following:

1. Aerosols

2. Foam agents

3. Fire retardants

4. Lubricants

In the making of how many of the above are hydrofluorocarbons used?

(a) Only one

(b) Only two

(c) Only three

(d) All four

Answer is (d) All four.

Hydrofluorocarbons (HFCs) are used in the making of all four mentioned substances:

  1. Aerosols: HFCs are commonly used as propellants in aerosol products, such as sprays for personal care, household cleaning, and automotive applications.
  2. Foam agents: HFCs are used as blowing agents in the production of foam materials, including foam insulation, foam packaging, and foam cushions.
  3. Fire retardants: HFCs are used in some fire extinguishing systems and fire retardant formulations.
  4. Lubricants: HFCs are used as lubricants in certain applications, especially in the aerospace industry.

Therefore, HFCs are used in the making of all four substances.

Q.31)  Consider the following statements:

1. Jhelum River passes through Wular Lake.

2. Krishna River directly feeds Kolleru Lake.

3. Meandering of the Gandak River formed Kanwar Lake.

How many of the statements given above are correct?

(a) Only one

(b) Only two

(c) All three

(d) None

.Ans: C

Statement 1 is Correct: The main source of water for Wular Lake is River Jhelum. This lake also has a small island in its centre called the ‘Zaina Lank’.      

Statement 2 is Correct: The Kolleru lake is a natural eutrophic lake, situated between the two major river basins of the Godavari and the Krishna, fed by two seasonal rivers and a number of drains and channels

Statement 3 is Correct: The Kanwar lake draws water from the confluence of the Gandak, the Bia and the Kareh river, is situated near Manjhaul, 22 km northwest of Begusarai, Bihar.

Q.32)  Consider the following pairs:

Port: Well known as

1. Kamarajar Port: First major port in India registered as a company

2. Mundra Port : Largest privately owned port in India

3. Visakhapatnam: Largest container port in India

How many of the above pairs are correctly matched?

(a) Only one pair

(b) Only two pairs

(c) All three pairs 

(d) None of the pairs

Ans: B

Statement 1 is Correct: Kamarajar Port, located on the Coromandel Coast about 24 km north of Chennai Port, Chennai, it is the 12th major port of India, and the first port in India which is a public company.

https://www.ennoreport.gov.in/content/#:~:text=it%20is%20the%2012th%20major%20port%20of%20India%2C%20and%20the%20first%20port%20in%20India%20which%20is%20a%20public%20company.

Statement 2 is Correct: The mega port at Mundra is a major economic gateway that caters to the northern hinterland of India with multimodal connectivity. The deep draft, all-weather port is the largest commercial port in India with state-of-the-art infrastructure.

Statement 3 is Incorrect: Mundra Port, India’s biggest commercial port by volumes, has overtaken state-run Jawaharlal Nehru Port Trust (JNPT) to become the country’s largest container gateway by handling 5.65 million twenty-foot equivalent units  in FY21.

Q.33)  Consider the following trees:

1. Jackfruit (Artocarpus heterophyllus) 

2. Mahua (Madhuca indica)

3. Teak (Tectona grandis)

How many of the above are deciduous trees?

(a) Only one

(b) Only two

(c) All three

(d) None

Ans: B

Statement 1 is Incorrect: Jackfruit is an evergreen tree characterised by its dense
foliage and exudation of latex from above ground plant parts including fruits when injured. https://krishi.icar.gov.in/jspui/bitstream/123456789/18834/1/Jackfruit%20paper%20AAB.pdf       

Statement 2 is Correct: Mahua tree is known as Indian Butter Tree. A medium sized to large deciduous tree with a large rounded crown

 Statement 3 Is Correct: Teak (Tectona grandis) is a tropical hardwood tree species in the family Lamiaceae. It is a large, deciduous tree that occurs in mixed hardwood forests. 

Q.34)  Consider the following statements:

1. India has more arable area than China.

2. The proportion of irrigated area is more in India as compared to China.

3. The average productivity per hectare in Indian agriculture is higher than that in China.

How many of the above statements are correct?

(a) Only one

(b) Only two

(c) All three

(d) None

Ans:B

Statement 1 is Correct     https://www.livemint.com/Politics/aI28vAplwqEqm2LnziWU3I/Theres-more-farmland-in-the-world-than-was-previously-thoug.html#:~:text=by%20Taboola-,India%20has%20the%20largest%20cropland%20of%20any%20country%20at%20179.8%20million%20hectares%2C%20compared%20with%20167.8%20million%20in%20the%20US%20and%20165.2%20million%20in%20China.,-Russia%20ranks%20fourth

Statement 2 is correct: . Irrigated area accounts for nearly 48.8 per cent of the 140 million hectare (mha) of agricultural land in India. China’s irrigation cover is 41% of cultivated area, and India’s is 48%.

Statement 3 is incorrect: China’s productivity in most crops is 50 to 100% higher than India’s


https://www.financialexpress.com/opinion/india-can-learn-agri-policy-lessons-from-china/1748398/

Q.35)  Which one of the following is the best example of repeated falls in sea level, giving rise to present-day extensive marshland?

(a) Bhitarkanika Mangroves 

(b) Marakkanam Salt Pans

(c) Naupada Swamp

(d) Rann of Kutch

Ans: D

The Rann of Kutch is a large seasonal salt marsh located in the Thar Desert in the state of Gujarat, India. It is a unique ecosystem characterized by vast stretches of saline marshland that gets flooded during the monsoon season and dries up during the rest of the year. The Rann of Kutch was formed due to multiple instances of sea level fluctuations and geological processes over millions of years, resulting in the deposition of salts and minerals in the region.

The other options mentioned, Bhitarkanika Mangroves, Marakkanam Salt Pans, and Naupada Swamp, are also wetland ecosystems but they do not exhibit the same characteristics of extensive marshland resulting from repeated falls in sea level like the Rann of Kutch.

Q.36)   Ilmenite and rutile, abundantly available in certain coastal tracts of India, are of which one of the

rich sources following?

(a) Aluminum

(b) Copper

(c) Iron

(d) Titanium

Ans: D

India is endowed with large resources of heavy minerals which occur mainly along coastal stretches of the country and also in inland placers.
Heavy mineral sands comprise a group of seven minerals, viz, ilmenite, leucoxene (brown ilmenite), rutile, zircon, sillimanite, garnet and monazite. Ilmenite (FeO.TiO2 ) and rutile (TiO2 ) are the two chief minerals of titanium.

Hence, option D is correct.
https://ibm.gov.in/writereaddata/files/08172015131610Ilmenite%20and%20Rutile.pdf

Q. 37) About three-fourths of world’s cobalt, a metal required for the manufacture of batteries for electric motor vehicles, is produced by –

(a) Argentina

(b) Botswana

(c) Democratic Republic of the Congo

(d) Kazakhstan

Ans: C

About three-fourths of the world’s cobalt, a metal required for the manufacture of batteries for electric motor vehicles, is produced by (c) Democratic Republic of the Congo.

The Democratic Republic of the Congo is the largest producer of cobalt globally, accounting for a significant portion of the world’s cobalt supply. Cobalt is a critical component in the production of lithium-ion batteries, which are widely used in electric vehicles and various electronic devices. The country’s rich reserves and favorable geological conditions make it a major player in the global cobalt market.

Q.38)  Which one of the following is a part of the Congo Basin?

(a) Cameroon

(b) Nigeria

(c) South Sudan

(d) Uganda

Ans: A

The Congo Basin spans across six countries—Cameroon, Central African Republic, Democratic Republic of the Congo, Republic of the Congo, Equatorial Guinea and Gabon.
Hence, option A is correct.

Q.39)  Consider the following statements:

1. Amarkantak confluence of Hills are at the Vindhya and Sahyadri Ranges.

2. Biligirirangan Hills constitute the easternmost part of Satpura Range.

3. Seshachalam Hills constitute the southernmost part of Western Ghats.

How many of the statements given above are correct?

(a) Only one

(b) Only two

(c) All three

(d) None

Ans: D

Amarkantak are at the confluence of the Vindhya and Satpura mountain ranges. Hence,

option 1 is not correct.

Seshachalam Hills are hilly ranges part of the Eastern Ghats in southern Andhra Pradesh state, in southeastern India.

The Biligirirangana Hills or Biligirirangan Hills (as referred to in biology and geology

is a hill range situated in south-western Karnataka, at its border with Tamil Nadu (Erode District).

Q.40) With reference to India’s projects connectivity, consider the following statements:

1. East-West Corridor under Golden Quadrilateral Project connects Dibrugarh and Surat.

2. Trilateral Highway connects Moreh in Manipur and Chiang Mai in Thailand via Myanmar.

3. Bangladesh-China-India-Myanmar Economic Corridor connects Varanasi in Uttar Pradesh with Kunming in China.

How many of the above statements are correct?

(a) Only one

(b) Only two

(c) All three

(d) None 

Ans: D

The East-West corridor of the Golden Quadrilateral connects Silchar and Porbandar cities in India. 

None of the statements provided are correct regarding India’s connectivity projects. The East-West Corridor under the Golden Quadrilateral Project connects Silchar in Assam to Porbandar in Gujarat, not Dibrugarh and Surat. The Trilateral Highway connects Moreh in Manipur, India, with Mae Sot in Thailand via Myanmar, not Chiang Mai. The Bangladesh-China-India-Myanmar Economic Corridor (BCIM EC) connects Kolkata in India with Kunming in China, not Varanasi.

https://www.thehindu.com/news/international/bangladesh-china-india-myanmar-bcim-economic-corridor-no-longer-listed-under-bri-umbrella/article26971613.ece

Q.41)  Consider the following statements: 

Statement-I: Interest income from the deposits (InvITs) distributed to their investors is in Infrastructure Investment Trusts exempted from tax, but the dividend is taxable.

Statement-II: InvITs are recognized as borrowers under the ‘Securitization and Recon- struction of Financial Assets and Enforcement of Security Interest Act, 2002’.

Which one of the following is correct in respect of the above statements? 

(a) Both Statement-I and Statement-II are correct and Statement-II is the correct explanation for Statement-I 

(b) Both Statement-I and Statement-II are correct and Statement-II is not the correct explanation for Statement-I

(c) Statement-I is correct but Statement-II is incorrect

(d) Statement-I -II is correct but Statement is incorrect

Answer: d

Statement-I: Interest income from the deposits (InvITs) distributed to their investors is in Infrastructure Investment Trusts exempted from tax, but the dividend is taxable: This statement is incorrect. In Infrastructure Investment Trusts (InvITs), both the interest income and dividend income distributed to the investors are taxable. There is no exemption from tax for interest income from InvITs.

Statement-II: InvITs are recognized as borrowers under the ‘Securitization and Reconstruction of Financial Assets and Enforcement of Security Interest Act, 2002’: This statement is incorrect. InvITs are not recognized as borrowers under the ‘Securitization and Reconstruction of Financial Assets and Enforcement of Security Interest Act, 2002’. This act primarily deals with the securitization and reconstruction of financial assets and enforcement of security interests in relation to loans and borrowings.

Based on the revised explanations, the correct answer is (d) Statement-I is incorrect, and Statement-II is incorrect. I apologize for the confusion caused by the earlier response.

https://www.civilsdaily.com/imp-must-read-rbis-faq-page-summary-for-prelims-2023/

Q.42)  Consider the following statements:

Statement-I : In the post-pandemic recent past many Central Banks worldwide had carried out interest rate hikes. 

Statement-II : Central Banks generally assume that they have the ability to counteract the rising consumer prices via monetary policy means. 

Which one of the following is correct in respect of the above statements? 

(a) Both Statement-I and Statement-II are correct and Statement-II is the correct explanation for Statement-I 

(b) Both Statement-I and Statement-II are correct and Statement-II is not the correct explanation for Statement-I 

(c) Statement-I is correct but  Statement-II is incorrect 

(d) Statement-I is incorrect but  Statement-II is correct

Answer: A

Statement-I: In the post-pandemic recent past, many Central Banks worldwide had carried out interest rate hikes: This statement is correct. In the post-pandemic period, as economies recover and inflationary pressures increase, some central banks have indeed raised interest rates to manage inflation and stabilize their economies. These interest rate hikes are implemented as part of monetary policy measures.

Statement-II: Central Banks generally assume that they have the ability to counteract the rising consumer prices via monetary policy means: This statement is the correct explanation for Statement-I. Central banks do assume that they have the ability to influence and control inflation through monetary policy tools, including interest rate adjustments. By raising interest rates, central banks aim to curb inflationary pressures by reducing borrowing and spending, thus addressing the rising consumer prices.

Based on the revised explanations, the correct answer is (a) Both Statement-I and Statement-II are correct, and Statement-II is the correct explanation for Statement-I.

Q.43)  Consider the following statements:

Statement-I: Carbon markets are likely to be one of the most widespread tools in the fight against climate change.

Statement-I1: Carbon markets transfer resources from the private sector to the State. 

Which one of the following is correct in respect of the above statements?

(a) Both Statement-I and Statement-II are correct and Statement-II is the correct explanation for Statement-I 

(b) Both Statement-I and Statement-II are correct and Statement-II is not the correct explanation for Statement-I

(c) Statement-I is correct but Statement-II is incorrect

(d) Statement-I is incorrect but Statement-II is correct

Answer : (a)

Statement-I: Carbon markets are likely to be one of the most widespread tools in the fight against climate change: This statement is correct. Carbon markets, such as emissions trading or cap-and-trade systems, have emerged as important tools in the fight against climate change. They provide economic incentives for reducing greenhouse gas emissions and promoting the transition to cleaner technologies.

Statement-II: Carbon markets transfer resources from the private sector to the State: This statement is the correct explanation for Statement-I. In carbon markets, private sector entities are typically required to purchase or acquire emission allowances or credits to cover their emissions. The revenue generated from the sale of these allowances or credits can be used by the State for various purposes, such as funding climate change mitigation and adaptation initiatives.

Based on the revised explanations, the correct answer is (a) Both Statement-I and Statement-II are correct, and Statement-II is the correct explanation for Statement-I. Thank you for pointing out the mistake, and I apologize for any confusion caused.

Q.44) Which one of the following activities of the Reserve Bank of India is considered to be part of ‘sterilization’?

(a) Conducting ‘Open Market Operations’ of settlement

(b) Oversight payment systems

(c) Debt and cash management for the Central and State Governments

(d) Regulating the functions of Non- banking Financial Institutions

Answer : (a)

Correct answer: (a) Conducting ‘Open Market Operations’ of settlement

‘Sterilization’ is an activity undertaken by the central bank to offset the impact of its interventions in the foreign exchange market on the domestic money supply. When the central bank intervenes in the foreign exchange market by buying foreign currency, it increases the domestic money supply. To prevent excess liquidity in the economy, the central bank conducts sterilization operations.

One of the key methods of sterilization is through ‘Open Market Operations’ (OMOs), specifically the sale or purchase of government securities. In the case of intervention to buy foreign currency, the central bank can sterilize the increased money supply by selling government securities in the open market, thereby reducing the money available in the economy. Conversely, if the central bank intervenes to sell foreign currency, it can sterilize the reduction in money supply by purchasing government securities.

Therefore, conducting ‘Open Market Operations’ of settlement is considered a part of ‘sterilization’ by the Reserve Bank of India.

Q.45) Consider the following marke

1. Government Bond Market

2. Call Money Market

3. Treasury Bill Market

4. Stock Market

How many of the above are included in capital markets?

(a) Only one

(b) Only two

(c) Only three

(d) All four

Answer : (b)

In the given options, only two of them are included in the capital markets:

  1. Government Bond Market: This market involves the buying and selling of government bonds, which are debt securities issued by the government to raise funds. It is an important segment of the capital market as it allows governments to borrow money from investors.
  2. Stock Market: The Stock Market, also known as the equity market or share market, is a market where shares or ownership interests in publicly-traded companies are bought and sold. It is a crucial component of the capital market, allowing companies to raise capital through the issuance of shares, and investors to trade in those shares.

The other two options, Call Money Market and Treasury Bill Market, are part of the money market rather than the capital market.

  1. Call Money Market: The Call Money Market is a segment of the money market where banks and financial institutions borrow and lend money for short durations, typically overnight.
  2. Treasury Bill Market: Treasury bills (T-bills) are short-term government securities with a maturity period of less than one year. The Treasury Bill Market is a part of the money market where these T-bills are traded.

Q.46)  Which one of the following best describes the concept of ‘Small Farmer Large Field’?

(a) Resettlement of a large number of people, uprooted from their countries due to war, by giving them a large cultivable land which they cultivate collectively and share the produce

(b) Many marginal farmers in an area organize themselves into groups and synchronize and harmonize selected agricultural operations

(c) Many marginal farmers in an area together make a contract with a corporate body and surrender. their land to the corporate body for a fixed term for which the corporate body makes a payment of agreed amount to the farmers

(d) A company extends loans, technical knowledge and material inputs to a number of small farmers in an area so that they produce the agricultural commodity required by the company for its manufacturing process and commercial production

Answer: (b)

The Small Farmers, Large Field (SFLF) model is founded on the same principles of aggregation and achieving economies of scale, through strengthening backward and forward integration along the supply chain and lowering costs by synchronizing key agricultural operations from field preparation to harvest.

https://indianexpress.com/article/india/agricultural-economics-how-doubling-of-farmers-income-is-possible-even-with-small-landholdings-5428084/#:~:text=The%20Small%20Farmers%2C%20Large%20Field%20%28SFLF%29%20model%20is,key%20agricultural%20operations%20from%20field%20preparation%20to%20harvest

Q.47)  Consider the following statements:

1. The Government of India provides Minimum Support Price for niger (Guizotia abyssinica) seeds.

2. Niger is cultivated as a Kharif crop.

3. Some tribal people in India use niger seed oil for cooking.

How many of the above statements are correct?

(a) Only one

(b) Only two

(c) All three

(d) None

Answer : (c) All three

Gond tribals of Satpura hill ranges in Madhya Pradesh traditionally grow niger on the top of the plateau, where water does not stagnate.But due amarbel (a twining parasitic plant known as Cuscuta chinensis in scientific lexicon) the productivity has declined rapidly due to which assured paddy cultivation has been trending. 

https://www.downtoearth.org.in/news/agriculture/illusive-oilseed-india-s-niger-seed-cultivation-is-declining-here-is-why-84380
https://pib.gov.in/PressReleasePage.aspx?PRID=1725612

Q.48)  Consider the investments in the following assets:

1. Brand recognition

2. Inventory

3. Intellectual property

4. Mailing list of clients

How many of the above are considered intangible investments?

(a) Only one

(b) Only two

(c) Only three

(d) All four

Answer : (c)  Only three 

The term inventory refers to the raw materials used in production as well as the goods produced that are available for sale. A company’s inventory represents one of the most important assets it has because the turnover of inventory represents one of the primary sources of revenue generation and subsequent earnings for the company’s shareholders. There are three types of inventory, including raw materials, work-in-progress, and finished goods. It is categorized as a current asset on a company’s balance sheet.

So Raw material is a tangible asset.Other three are intangible.

Q.49)  Consider the following:

1. Demographic performance 

2. Forest and ecology

3. Governance reforms 

4.Stable government

5. Tax and fiscal efforts

For the horizontal tax devolution, the Fifteenth Finance Commission used how many of the above as criteria other than population area and income distance?

(a) Only two

(b) Only three 

(c) Only four 

(d) All five

Answer : (b)  Only three 

Governance reforms and Stable governments are not considered under horizontal tax devolution criteria.

Q.50)  Consider the following infrastructure sectors:

1. Affordable housing

2. Mass rapid transport 

3. Health care

4. Renewable energy

How many of the above does UNOPS Sustainable Investments in Infrastructure and Innovation (S3i) initiative focus for its investments?

(a) Only one

(b) Only two

(c) Only three

(d) All four

Answer: (d) All four

The UNOPS Sustainable Investments in Infrastructure and Innovation (S3i) initiative focuses on investments in all four infrastructure sectors mentioned:

  1. Affordable housing: The S3i initiative aims to invest in projects that promote affordable housing, particularly in regions where there is a significant housing shortage or lack of adequate housing options.
  2. Mass rapid transport: The S3i initiative aims to invest in projects related to mass rapid transport systems, such as metro railways, light rail systems, or other forms of efficient and sustainable public transportation.
  3. Health care: The S3i initiative also focuses on investments in healthcare infrastructure, including the development of hospitals, clinics, and healthcare facilities, especially in underserved areas.
  4. Renewable energy: Renewable energy is another sector that the S3i initiative focuses on for its investments. It aims to support projects related to renewable energy generation, such as solar power plants, wind farms, and other sustainable energy initiatives.

Therefore, the UNOPS S3i initiative focuses on investments in all four infrastructure sectors: affordable housing, mass rapid transport, healthcare, and renewable energy.

Q.51)  With reference to Home Guards, consider the following statements:

1. Home Guards are raised under the Home Guards Act and Rules of the Central Government.

2. The role of the Home Guards is to serve as an auxiliary force to the police in maintenance of internal security.

3. To prevent infiltration on the international border/coastal areas, the Border Wing Home Guards Battalions have been raised in some States.

How many of the above statements are correct?

(a) Only one

(b) Only two

(c) All three

(d) None

Answer : (b)  Only two.

Here first statement is wrong as Home Guards are raised under the Home Guards Act and Rules of the States/Union Territories.

Fifteen Border Wing Home Guards (BWHG) Battalions have been raised in the border States viz. Punjab (6 Bns.), Rajasthan ( 4 Bns.), Gujarat (2 Bns.) and one each Battalion for Meghalaya, Tripura and West Bengal to serve as an auxiliary to Border Security Force for preventing infiltration on the international border/coastal areas, guarding of VA/VPs and lines of communication in vulnerable area at the time of external aggression.

Q.52)  With reference to India, consider the following pairs :

Action :  Act under which it is covered

1. Unauthorized wearing of police or military uniforms : The Official Secrets Act, 1923

2. Knowingly misleading or otherwise interfering with a police officer or military officer when engaged in their duties : The Indian Evidence Act, 1872  

3. Celebratory gunfire that endanger personal safety of others : The Arms (Amend- ment) Act, 2019

How many of the above pairs are correctly matched?

(a) Only one

(b) Only two

(c) All three

(d) None

Answer : (b)  Only two

Out of the given pairs, two of them are correctly matched:

1. Unauthorized wearing of police or military uniforms: The unauthorized wearing of police or military uniforms is covered under The Official Secrets Act, 1923. This act prohibits unauthorized persons from wearing the uniforms of police or military personnel.

2. Knowingly misleading or otherwise interfering with a police officer or military officer when engaged in their duties: This action is not covered under The Indian Evidence Act, 1872. The Indian Evidence Act primarily deals with the rules of evidence in Indian courts and does not specifically address misleading or interfering with police or military officers.

3. Celebratory gunfire that endangers the personal safety of others: This action is covered under The Arms (Amendment) Act, 2019. The act regulates the possession, use, and sale of firearms and ammunition, and celebratory gunfire that endangers the safety of others would be subject to its provisions.

Based on the revised explanations, the correct answer is (b) Only two.

Q.53) Consider the following pairs:

Regions often mentioned in news:: Reason for being in news

1. North Kivu and Ituri: War between Armenia and Azerbaijan

2. Nagorno-Karabakh : Insurgency in Mozambique

3. Kherson and Zaporizhzhia : Dispute between Israel and Lebanon

How many of the above pairs are correctly matched?

(a) Only one

(b) Only two

(c) All three

(d) None

Correct Answer:(d) None of the above

https://www.civilsdaily.com/1-year-russia-ukraine-war/  (Refer the map within)

Q.54)  Consider the following statements: 

Statement-I : Israel has established diplomatic relations with some Arab States.

Statement-II: The ‘Arab Peace Initiative’ mediated by Saudi Arabia was signed by Israel and Arab League.

Which one of the following is correct in respect of the above statements? 

(a) Both Statement-I and Statement-II are correct and Statement-II is the 515 correct explanation for Statement-I 

(b) Both Statement-I and Statement-II are correct and Statement-II is not the correct explanation for Statement-I

(c) Statement-I is correct but Statement-II is incorrect

(d) Statement-I is incorrect but Statement-II is correct

Answer:(c)

Israel has not signed the Arab Peace Initiative.

Q.55)  Consider the following pairs with regard to sports awards:

1. Major Dhyan Chand :For Khel Ratna Award the Most spectacular And outstanding performance 

2. Arjuna Award sportsperson over period of last four years : For the lifetime achievement by a sportsperson

3. Dronacharya Award: To honor eminent coaches who have successfully trained sportspersons teams

4. Rashtriya Khel Protsahan Puraskar : To recognize the contribution made by sports persons even after retirement

How many of the above pairs are correctly matched?

(a) Only one

(b) Only two

(c) Only three

(d) All four

Answer: (b) Only Two

Major Dhyan Chand Khel Ratna Award is the highest sporting award given by the Ministry of Youth Affairs and Sports for the spectacular and most outstanding performance in the field of sports by a sportsperson over a period of four years.

Q.56) Consider the following statements in respect of the 44th Chess Olympiad, 2022:

1. It was the first time that the Chess Olympiad was held in India.

2. The official mascot was named Thambi’.

3. The trophy for the winning team in the open section is the Vera Menchik Cup.

4. The trophy for the winning team in the women’s section is the Hamilton-Russell Cup.

How many of the statements given above are correct?

(a) Only one

(b) Only two

(c) Only three

(d) All four

Correct Answer:(b) Only two

First two statememts are correct.Thamb means “hey little brother” in the native language(Tamil Nadu), a phrase that is commonly used to address someone.Here it was represented by Brown Horse

Last two statements altered.Vera Menchik cup was for womans section and Hamiltion Russell cup for open section.

Q.57)  Consider the following pairs:

Area of conflict mentioned in news:: Country where it is located

1. Donbas : Syria

2. Kachin: Ethiopia

3. Tigray: North Yemen

How many of the above pairs are correctly matched?

(a) Only one

(b) Only two

(c) All three

(d) None

Answer:(d) None

The Donbas or Donbass is a historical, cultural, and economic region in eastern Ukraine. Parts of the Donbas are occupied by Russia as a result of the Russo-Ukrainian War. The word Donbas is a portmanteau formed from “Donets Basin”, an abbreviation of “Donets Coal Basin” 

The area of Kachin State is 89,041 km 2 (34,379 sq mi). The capital of the state is Myitkyina. Other important towns include Bhamo, Mohnyin and Putao. Kachin State has Myanmar’s highest mountain, Hkakabo Razi at 5,889 meters (19,321 ft), forming the southern tip of the Himalayas, and a large inland lake.

The Tigray Region, officially the Tigray National Regional State, is the northernmost regional state in Ethiopia. The Tigray Region is the homeland of the Tigrayan, Irob, and Kunama people. Its capital and largest city is Mekelle.

Q.58)  In recent years Chad, Guinea, Mali and Sudan caught the international attention for which one of the following reasons common to all of them?

(a) Discovery of rich deposits of rare earth elements

(b) Establishment of Chinese military bases

(c) Southward expansion of Sahara Desert

(d) Successful coups

Answer : (d)

In the past 18 months, military leaders have toppled the governments of Mali, Chad, Guinea, Sudan and Burkina Faso.

Q.59)  Consider the following heavy industries:

1. Fertilizer plants

2. Oil refineries

3. Steel plants

Green hydrogen is expected to play a significant role in decarbonizing how many of the above industries?

(a) Only one

(b) Only two

(c) All three

(d) None

Correct answer:(c) All three

Green hydrogen featured in a number of emissions reduction pledges at the UN Climate Conference, COP26, as a means to decarbonize heavy industry, long haul freight, shipping, and aviation. Governments and industry have both acknowledged hydrogen as an important pillar of a net zero economy.

Green hydrogen for ammonia synthesis in fertilizer manufacturing can support the net-zero vision for India.

Refineries, which turn crude oil into fuel such as petrol and diesel, use hydrogen to lower the sulfur content of diesel fuel. This hydrogen is currently produced using fossil fuels such as natural gas. IOC plans to use electricity generated from renewable sources such as solar to split water to produce green hydrogen.

Green hydrogen can be used in steel production in two ways1. It can be used as an alternative injection material to pulverized coal injection (PCI) to improve the performance of conventional blast furnaces. The use of green hydrogen can reduce carbon emissions in blast furnaces by 20% However, carbon neutral steel is not produced as the blast furnace still uses coal as a reducing agent.Green hydrogen can also be produced via electrolysis using just water and renewable electricity, making it completely free of CO₂ emissions. The green steel method uses hydrogen to reduce iron pellets into sponge iron, metallic iron that can then be processed to form steel.

https://www.reuters.com/world/india/india-require-refiners-fertiliser-plants-use-some-green-hydrogen-2021-08-09/

Q.60)  Consider the following statements about G-20:

1. The G-20 group  was originally established as a platform for the Finance Ministers and Central Bank Governors to discuss international economic and financial issues.

2. Digital public infrastructure is one of India’s G-20 priorities.

Which of the statements given above is/are correct?

(a) 1 only

(b) 2 only

(c) Both 1 and 2

(d) Neither 1 nor 2

Answer:(c) Both 1 and 2

India and the G20 Presidency: Its Priorities and Challenges – India Foundation 

Q.61)  With reference to Indian History, Alexander Rea, A. H. Longhurst, Robert Sewell, James Burgess and Walter Elliot were associated with

(a) archaeological excavations

(b) establishment of English Press in Colonial India

(c) establishment of Churches in Princely States

(d) construction of railways in Colonial India

Answer :  (a)

Explanation: They were associated with the various archaeological excavations.

Q.62)  Consider the following pairs:

Site: Well known for

1. Besnagar : Shaivite cave shrine

2. Bhaja : Buddhist cave shrine

3. Sittanavasal: Jain cave shrine

How many of the above pairs are correctly matched?

(a) Only one

(b) Only two

(c) All three

(d) None

Answer :  B

Explanation: Udaigiri represents Shaiva while Besnagar represents Vasudeva, also treated as lord of the lords. Hence pair 1 is incorrectly matched.

Bhaja Caves are the best example of Buddhist cave architecture in India. They are located on a hill village Bhaja in Pune district near Lonavala, Mumbai. Hence pair 2 is correctly matched. 

Sittanavasal cave temple was built by Pallava King Mahendravarma (580–630 AD) prior to his conversion from Jainism to Hinduism. Hence pair 3 is correctly matched.

Q.63)  Consider the following statements:

Statement-I: 7th August is declared as the National Handloom Day. 

Statement-II : It was in 1905 that the Swadeshi Movement was launched on the same

day.

Which one of the following is correct in respect of the above statements? 

(a) Both Statement-I and Statement-II are correct and Statement-II is the correct explanation for Statement-I

(b) Both Statement-I and Statement-II are correct and Statement-II is not the correct explanation for Statement-I

(c) Statement-I is correct Statement-II is incorrect

(d) Statement-I is incorrect Statement-II is correct

Answer : (a)

Both the statements are correct.

National Handloom Day is commemorated in India on August 7 each year.

August 7 was designated to celebrate National Handloom Day to memorialize the ‘Swadeshi’ Movement. Therefore, there is a strong connection between National Handloom Day and the Swadeshi Movement.

Q.64)  Consider the following statements in respect of the National Flag of India according to the Flag Code of India, 2002:

Statement-I : One of the standard sizes of the National  Flag of India is 600 mm x 400 mm.

Statement-II: The ratio of the length to the height (width) of the Flag shall be 3:2. 

Which one of the following is correct in respect of the above statements? 

(a) Both Statement-I and Statement-II are correct and Statement-II is the correct explanation for Statement-I

(b) Both Statement-I and Statement-II are correct and Statement-II is not the correct explanation for Statement-I

(c) Statement-I is correct Statement-II is incorrect

(d) Statement-I is incorrect Statement-II is correct

Answer :  (d)

The ratio of the length to width should be in the ratio of 3:2 and as per the given below table. 600 x 400 is not mentioned. Hence statement 1 is incorrect while 2 is correct.

Q.65)  Consider the following statements in respect of the Constitution Day:

Statement-I: Constitution Day is celebrated on 26th November every year to promote constitutional values among citizens. 

Statement-II: On 26th November, 1949, the Constituent Assembly of India set up a Drafting Committee under the Chairmanship of Dr. B. R. Ambedkar to prepare a Draft Constitution of India.

Which one of the following is correct in respect of the above statements? 

(a) Both Statement-I and Statement-II are correct and Statement-II is the correct explanation for Statement-I

(b) Both Statement-I and Statement-II are correct and Statement-II is not the correct explanation for Statement-I

(c) Statement-I is correct Statement-II is incorrect

(d) Statement-I is incorrect Statement-II is correct

Answer :  (c)

Constitution Day, also known as “National Law Day”, is celebrated in India on 26 November every year to commemorate the adoption of the Constitution of India. The Ministry of Social Justice and Empowerment on 19th November 2015 notified the decision of Government of India to celebrate the 26th day of November every year as ‘Constitution Day’ to promote Constitution values among citizens. Hence statement 1 is correct.

On 26th November, 1949 our constitution was adopted. Hence statement 2 is incorrect.

Q.66)  Consider the following statements:  

Statement-I: Switzerland is one of the leading exporters of gold in terms of value. 

Statement-II: Switzerland has the second largest gold reserves in the world.

Which one of the following is correct in respect of the above statements? 

(a) Both Statement-I and Statement-II are correct and Statement-II is the correct explanation for Statement-I

(b) Both Statement-I and Statement-II are correct and Statement-II is not the correct explanation for Statement-I

(c) Statement-I is correct Statement-II is incorrect

(d) Statement-I is incorrect Statement-II is correct

Answer :  C 

According to several organisations, Switzerland is consistently the world’s leading gold exporting country based on value. Hence statement 1 is correct.

According to several organisations, USA has the highest gold reserves followed by Germany. Hence statement 2 is incorrect.

Q.67)  Consider the following statements:

Statement-I: Recently, the United States of America (USA) and the European Union (EU) have launched the Trade and Technology Council’.

Statement-II: The USA and the EU claim that through this they are trying to bring technological progress and physical productivity under their control. 

Which one of the following is correct in respect of the above statements? 

(a) Both Statement-I and Statement-II are correct and Statement-II is the correct explanation for Statement-l 

(b) Both Statement-I and Statement-II are correct and Statement-II is not the correct explanation for Statement-I

(c) Statement-I is correct but Statement-II is incorrect

(d) Statement-I is incorrect but Statement-II is correct

Answer : C

The Trade and Technology Council is a transatlantic political body which serves as a diplomatic forum to coordinate technology and trade policy between the United States and European Union. Hence statement 1 is correct.

Through the Council, the EU and the US are working together to: ensure that trade and technology serve our societies and economies, while upholding our common values. strengthen our technological and industrial leadership. Hence statement 2 is incorrect.

Q.68) Consider the following statements :

Statement-I: India accounts for 3.2% of global export of goods.

Statement-II : Many local companies and some foreign companies operating in India took advantage of India’s ‘Production-linked Incentive’ scheme. 

Which one of the following is correct in respect of the above statements? 

(a) Both Statement-I and Statement-II are correct and Statement-II is the correct explanation for Statement-I 

(b) Both Statement-I and Statement-II are correct and Statement-II is not the correct explanation for Statement-I

(c) Statement-I is correct but Statement-II is incorrect

(d) Statement-I is incorrect but Statement-II is correct

Answer :  (d)

India aims to raise the share of its exports in global trade to 3% by 2027 which is less than 2% right now. Hence statement 1 is incorrect.

Many local and foreign enterprises have taken advantage of the PLI scheme. Hence statement 2 is correct.

Q.69)  Consider the following statements:

The ‘Stability and Growth Pact’ of the European Union is a treaty that

1. limits the levels of the budgetary deficit of the countries of the European Union

2. makes the countries of the European Union to share their infrastructure facilities

3. enables the countries of the European Union to share their technologies

How many of the above statements are correct?

(a) Only one

(b) Only two

(c) All three

(d) None

 Answer : (a)

The Stability and Growth Pact (SGP) is an agreement, among all of the 27 member states of the European Union, to facilitate and maintain the stability of the Economic and Monetary Union (EMU). The purpose of the pact was to ensure that fiscal discipline would be maintained and enforced in the EMU. It deals with the fiscal discipline of EU members. Hence only statement 1 is correct.

Q.70)  Consider the following statements:

1. Recently, all the countries of the United Nations have adopted the first-ever compact for international migration, the ‘Global Compact for Safe, Orderly and Regular Migration (GCM)’.

2. The objectives and commitments stated in the GCM are binding on the UN member countries. 

3. The GCM addresses internal migration or internally displaced people also in its objectives and commitments.

How many of the above statements are correct?

(a) Only one

(b) Only two

(c) All three

(d) None

Answer :  D

Signed by 164 members for adoption, Global Compact for Safe, Orderly and Regular Migration creates a non-legally binding and co-operative framework that builds on commitments agreed upon by member states in the New York Declaration for refugees and migrants of December 2016. Hence statements 1 and 2 are incorrect.

It includes all types of international migrations but not internal. Hence statement 3 is incorrect.

Q.71)   Consider the following countries: 

1. Bulgaria 

2. Czech Republic 

3. Hungary 

4. Latvia 

5. Lithuania 

6. Romania 

How many of the above-mentioned countries share a land border with Ukraine? 

(a) Only two 

(b) Only three 

(c) Only four 

(d) Only five

Answer :  A

Ukraine is bordered by Belarus to the north, Russia to the east, the Sea of Azov and the Black Sea to the south, Moldova and Romania to the southwest, and Hungary, Slovakia, and Poland to the west.

https://www.civilsdaily.com/1-year-russia-ukraine-war/ (Refer the map)

Q.72)  With reference to the Earth’s atmosphere, which one of the following statements is correct?

(a) The total amount of insolation received at the equator is roughly about 10 times that received at the poles.

(b) Infrared rays constitute roughly two-thirds of insolation.

(c) Infrared waves are largely absorbed by water vapour in a concentrated atmosphere.

(d) Infrared waves are a part of the lower visible spectrum of electromagnetic waves of solar radiation.

Answer :  B

Infrared waves, or infrared light, are part of the electromagnetic spectrum. People encounter Infrared waves every day; the human eye cannot see it, but humans can detect it as heat. Hence option d is not correct.

When the solar radiation passes through the atmosphere, water vapour, ozone and other gases absorb much of the near infrared radiation. 

Infrared makes up 49.4%, visible 42.3% and ultraviolet 8% of the total solar radiation. Hence option B is incorrect.

Q.73)  Consider the following statements:

Statement-I: The soil in tropical rain forests is rich in nutrients.

Statement-II: The high temperature and moisture of rain forests cause tropical dead organic matter in the soil to decompose quickly.

Which one of the following is correct in respect of the above statements? 

(a) Both Statement-I and Statement-II are correct and Statement-II is the correct explanation for Statement-I 

(b) Both Statement-I and Statement-II are correct and Statement-II is not the correct explanation for Statement-I

(c) Statement-I is correct Statement-II is incorrect

(d) Statement-I but is incorrect but Statement-II is correct

Answer : (d)

Explanation:

Statement-I: Contrary to what many might expect, the soil in tropical rainforests is generally not rich in nutrients. This is because while these forests are lush and support a great diversity of plant life, the high rainfall typical of these ecosystems can leach nutrients from the soil, making it relatively poor in nutrients.

Statement-II: This statement is correct. The high temperature and moisture of tropical rainforests do indeed promote the rapid decomposition of dead organic matter. This process returns nutrients quickly to the ecosystem, where they are rapidly taken up by plants and other organisms, contributing to the richness and diversity of life in these forests.

Therefore, Answer is (d) Statement-I is incorrect but Statement-II is correct.

Q.74)  Consider the following statements: 

Statement-I: The temperature contrast between continents and oceans is greater during summer than in winter.

Statement-II : The specific heat of water is more than that of land surface.

Which one of the following is correct in respect of the above statements?  

(a) Both Statement-I and Statement-II are correct and Statement-II is the correct explanation for Statement-I 

(b) Both Statement-I and Statement-II are correct and Statement-II is not the correct explanation for Statement-I 

(c) Statement-I is correct but Statement-II is incorrect 

(d) Statement-I is incorrect but Statement-II is correct

Answer: (a) Both Statement-I and Statement-II are correct, and Statement-II is the correct explanation for Statement-I

Statement-I: The temperature contrast between continents and oceans is greater during summer than in winter: This statement is correct. During summer, land surfaces heat up more quickly than oceans due to differences in their heat capacities. This leads to warmer temperatures over land, creating a greater temperature contrast between continents and oceans.

Statement-II: The specific heat of water is more than that of the land surface: This statement is correct. Water has a higher specific heat capacity compared to land. Specific heat is the amount of heat required to raise the temperature of a substance by a certain amount. Due to water’s higher specific heat capacity, it can absorb and store more heat energy compared to land. As a result, water takes longer to heat up and cool down compared to land, contributing to the temperature contrast between continents and oceans.

Statement-II provides the correct explanation for Statement-I. The higher specific heat capacity of water is the reason why land surfaces heat up more quickly in summer, leading to a greater temperature contrast between continents and oceans.

Q.75)  Consider the following statements:

1. In a seismograph, P waves are recorded earlier than S waves.

2. In P waves, the individual particles vibrate to and fro in the direction of wave propagation whereas in S waves, the particles vibrate up and down at right angles to the direction of wave propagation.

Which of the statements given above is/are correct?

(a) 1 only

(b) 2 only

(c) Both 1 and 2 

(d) Neither 1 nor 2

Answer :  (c)

Explanation:

Both statements are correct.

P waves (Primary waves) are the fastest kind of seismic wave and the first to arrive at seismographs during an earthquake, hence the name “Primary”. They are followed by S waves (Secondary waves), which are slower and arrive after the P waves.

P waves are compressional or longitudinal waves, meaning the ground particles vibrate in the same direction as the wave propagation. In contrast, S waves are shear or transverse waves, causing the ground particles to vibrate perpendicular to the direction of wave propagation (up and down or side to side).

So the Answer: (c) Both 1 and 2 are correct.

Q.76)  With reference to coal-based thermal power plants in India, consider the following statements :

1. None of them uses seawater.

2. None of them are set up in water-stressed districts.

3. None of them is privately owned.

How many of the above statements are correct?

(a) Only one

(b) Only two

(c) All three

(d) None

Answer : (d) None

  1. None of the coal-based thermal power plants in India uses seawater: This statement is incorrect. Some coal-based thermal power plants in India do use seawater for cooling purposes.
  2. None of the coal-based thermal power plants in India is set up in a water-stressed district: This statement is incorrect. There are coal-based thermal power plants in India that are located in water-stressed districts. The availability of water for cooling and other operational requirements is an important consideration in the location of power plants, but it does not necessarily mean that all plants are set up only in areas with abundant water resources.
  3. None of the coal-based thermal power plants in India is privately owned: This statement is incorrect. There are privately owned coal-based thermal power plants in India, in addition to those owned by government or public sector entities.

Q.77)  ‘‘Wolbachia method’ is sometimes talked about with reference to which one of the following?

(a) Controlling the viral diseases spread by mosquitoes

(b) Converting crop residues into packing material

(c) Producing biodegradable plastics 

(d) Producing biochar from thermo- chemical conversion of biomass

Answer : (a) Controlling the viral diseases spread by mosquitoes

The “Wolbachia method” is a biological control method used to control the spread of viral diseases, such as dengue, Zika, and chikungunya, which are transmitted by mosquitoes. Wolbachia is a naturally occurring bacterium found in many insects, including mosquitoes. When male mosquitoes carrying Wolbachia are released and mate with wild female mosquitoes, the resulting eggs do not hatch or develop properly, reducing the mosquito population over time. This method has shown promising results in reducing the transmission of viral diseases by mosquitoes and has been implemented in various regions as a strategy for mosquito-borne disease control.

Q.78)  Consider the following activities: 

1. Spreading finely ground basalt rock on farmlands extensively

2. Increasing the alkalinity of oceans by adding lime

3. Capturing carbon dioxide released by various industries and pumping it into abandoned subterranean mines in the form of carbonated waters

How many of the above activities are often considered and discussed for carbon capture and sequestration?

(a) Only one

(b) Only two

(c) All three

(d) None

Answer : (c) All three

  1. Spreading finely ground basalt rock on farmlands extensively: This activity, known as enhanced weathering, involves spreading finely ground basalt rock on farmlands to increase the rate of carbon dioxide removal from the atmosphere through chemical reactions. The basalt reacts with carbon dioxide to form carbonate minerals, effectively storing carbon in a solid form.
  2. Increasing the alkalinity of oceans by adding lime: This activity, known as ocean alkalinity enhancement, involves adding lime or other alkaline substances to the oceans to increase their alkalinity. This process promotes the absorption of carbon dioxide from the atmosphere into the ocean, enhancing the ocean’s capacity to act as a carbon sink.
  3. Capturing carbon dioxide released by various industries and pumping it into abandoned subterranean mines in the form of carbonated waters: This activity refers to carbon capture and storage (CCS), which involves capturing carbon dioxide emissions from industrial processes and storing them underground in suitable geological formations, such as depleted oil and gas reservoirs or deep saline aquifers.

All three activities mentioned are often considered and discussed as potential methods for carbon capture and sequestration to mitigate climate change by reducing carbon dioxide levels in the atmosphere. Therefore, the correct answer is (c) All three.

Q.79)  ‘Aerial metagenomics’ best refers to which one of the following situations? 

(a) Collecting DNA samples from air in a habitat at one go

(b) Understanding the genetic makeup of avian species of a habitat

(c) Using air-borne devices to collect blood samples from moving animals

(d) Sending drones to inaccessible areas to collect plant and animal samples from land surfaces and water bodies

Answer: (a)

The best interpretation of “aerial metagenomics” refers to option (A) – collecting DNA samples from the air in a habitat at one go. Aerial metagenomics involves the sampling and analysis of genetic material, such as DNA, found in the air. It aims to characterize and study the microbial communities present in the air, including bacteria, viruses, fungi, and other microorganisms. This technique allows scientists to study the biodiversity and genetic composition of airborne organisms without the need for direct sampling from individual organisms.

Q80)   ‘Microsatellite DNA’ is used in the case of which one of the following?

(a) Studying the evolutionary relationships among various species of fauna

(b) Stimulating ‘stem cells’ to transform into diverse functional tissues

(c) Promoting clonal propagation of horticultural plants

(d) Assessing the efficacy of drugs by conducting series of drug trials in a population

Answer: (a)

The term “Microsatellite DNA” (also known as “short tandem repeats” or “microsatellites”) is primarily used in the case of option (a) – studying the evolutionary relationships among various species of fauna. Microsatellites are short, repetitive sequences of DNA found in the genomes of many organisms, including animals, plants, and humans.

Microsatellite DNA is characterized by tandem repeats of short DNA sequences, typically 1-6 base pairs in length. These repeats can vary in the number of times they are repeated, creating genetic variation among individuals. By analyzing the variations in microsatellite regions across different species, scientists can study the evolutionary relationships, genetic diversity, and population structure of organisms.

Microsatellites are particularly useful in population genetics studies, where they can provide insights into gene flow, genetic drift, and relatedness among individuals or populations. They can be used to determine the genetic relatedness between different species or within a single species, assess population sizes and migration patterns, and investigate patterns of genetic adaptation and speciation.

While microsatellite DNA has various applications in genetic research, it is specifically associated with the study of evolutionary relationships and population genetics, making option (a) the correct choice in this case.

https://economictimes.indiatimes.com/magazines/panache/the-dynamic-dna-thank-your-parents-for-hair-skin-eye-colour-even-the-fingerprints/articleshow/97916166.cms?from=mdr

Q81)   Consider the following statements in relation to Janani Suraksha Yojana: 

1. It is a safe motherhood intervention of the State Health Departments. 

2. Its objective is to reduce maternal and neonatal mortality among poor pregnant women.

3. It aims to promote institutional delivery among poor pregnant women.

4. Its objective includes providing public health facilities to sick infants up to one year of age.

How many of the statements given above are correct?

(a) Only one

(b) Only two

(c) Only three

(d) All four

Answer :  (B)

  1. It is a safe motherhood intervention of the State Health Departments: This statement is not entirely correct. The Janani Suraksha Yojana (JSY) is not specifically a safe motherhood intervention of the State Health Departments. It is a centrally sponsored scheme implemented by the Government of India, aiming to improve maternal and neonatal health outcomes across the country. While the State Health Departments play a role in implementing the program at the state level, it is not limited to their intervention alone.
  2. Its objective is to reduce maternal and neonatal mortality among poor pregnant women: This statement is correct. One of the primary objectives of the Janani Suraksha Yojana is to reduce maternal and neonatal mortality rates, particularly among women from economically disadvantaged backgrounds. The scheme provides financial assistance and incentives to encourage poor pregnant women to deliver their babies in healthcare institutions, thereby increasing access to skilled care during childbirth and reducing the risks associated with home deliveries.
  3. It aims to promote institutional delivery among poor pregnant women: This statement is correct. One of the main goals of the Janani Suraksha Yojana is to promote institutional deliveries among poor pregnant women. The scheme provides financial support to pregnant women who choose to deliver their babies in government healthcare facilities, accredited private hospitals, or public health centers. By encouraging institutional deliveries, the scheme aims to ensure that pregnant women receive proper antenatal, intranatal, and postnatal care, reducing the risk of complications and improving maternal and neonatal health outcomes.
  4. Its objective includes providing public health facilities to sick infants up to one year of age: This statement is not correct. The Janani Suraksha Yojana primarily focuses on maternal and neonatal health during childbirth and does not specifically address the provision of public health facilities to sick infants up to one year of age. There may be other government programs or initiatives aimed at providing healthcare facilities and services for sick infants, but it is not a direct objective of JSY.

Therefore, based on the explanations provided, the Answer :  is (b) Only two.

Q.82)  Consider the following statements in the context of interventions being undertaken under Anaemia Mukt Bharat Strategy:

1. It provides prophylactic calcium supplementation for pre-school children, adolescents and pregnant Women.

2. It runs a campaign for delayed Cord clamping at the time of child birth.

3. It provides for periodic deworming to children and adolescents

4. It addresses non-nutritional causes of anaemia in endemic pockets with special focus on malaria, hemoglobinopathies and fluorosis.

How many of the statements given above are correct?

(a) Only one

(b) Only two

(c) Only three

(d) All four

Answer: (c) Only three

  1. It provides prophylactic calcium supplementation for pre-school children, adolescents, and pregnant women: This statement is correct. The Anaemia Mukt Bharat (AMB) Strategy includes the provision of prophylactic calcium supplementation for specific target groups, including pre-school children, adolescents, and pregnant women. Calcium supplementation is important for maintaining healthy bone development and preventing anaemia, especially during growth phases and pregnancy.
  2. It runs a campaign for delayed cord clamping at the time of childbirth: This statement is not correct. The Anaemia Mukt Bharat Strategy primarily focuses on interventions to prevent and address anaemia in the population. While delayed cord clamping may have benefits for newborns, such as increased iron stores, it is not specifically mentioned as a campaign or intervention under the AMB Strategy.
  3. It provides for periodic deworming to children and adolescents: This statement is correct. The Anaemia Mukt Bharat Strategy recognizes the importance of deworming as a preventive measure against anaemia. Periodic deworming of children and adolescents helps to control parasitic infections, such as soil-transmitted helminths, which can contribute to anaemia.
  4. It addresses non-nutritional causes of anaemia in endemic pockets with a special focus on malaria, hemoglobinopathies, and fluorosis: This statement is correct. The Anaemia Mukt Bharat Strategy acknowledges that anaemia can have non-nutritional causes, particularly in specific regions or pockets where certain conditions prevail. These conditions include malaria (which can cause hemolytic anaemia), hemoglobinopathies (genetic disorders affecting hemoglobin production), and fluorosis (excessive fluoride intake leading to anaemia). The strategy emphasizes addressing these specific causes of anaemia in endemic areas.

Based on the explanations provided, the Answer :  is (c) Only three.

Q.83) Consider the following statements:

1. Carbon fibers are used in the Manufacture of components used in automobiles and aircrafts.

2. Carbon fibers once used cannot be recycled.

Which of the statements given above is/are correct?

(a) 1 only

(b) 2 only

(c) Both 1 and 2

(d) Neither 1 nor 2


Answer: (a) 1 only

  1. Carbon fibers are used in the manufacture of components used in automobiles and aircraft: This statement is correct. Carbon fibers have excellent strength-to-weight ratio and are widely used in various industries, including automotive and aerospace. They are used to manufacture components such as body panels, frames, wings, and other structural parts in automobiles and aircraft due to their lightweight and high strength properties.
  2. Carbon fibers once used cannot be recycled: This statement is incorrect. Carbon fibers can be recycled, although the recycling process is more complex compared to other materials. The recycling of carbon fiber composites involves methods such as pyrolysis or solvent-based processes to break down the resin matrix and recover the carbon fibers. Recycling carbon fibers helps to reduce waste and environmental impact by reusing the valuable carbon fiber material.

Therefore, only statement 1 is correct, and the Answer :  is (a) 1 only.

Q.84) Consider the following actions:

1. Detection of car crash/collision which results in the deployment of airbags almost instantaneously

2. Detection of accidental free fall of a laptop towards the ground which results in the immediate turning off of the hard drive

3. Detection of the tilt of the mart phone which results in the rotation of display between portrait and landscape mode.

In how many of the above actions is the function of accelerometer required?

(a) Only one

(b) Only two

(c) All three

(d) None

Answer: (c) All three

  1. Detection of car crash/collision which results in the deployment of airbags almost instantaneously: Accelerometers are used in vehicles to detect sudden changes in acceleration, such as during a car crash or collision. The accelerometer can sense the rapid deceleration or impact force and trigger the deployment of airbags to protect the occupants. So, the function of an accelerometer is required in this action.
  2. Detection of accidental free fall of a laptop towards the ground which results in the immediate turning off of the hard drive: Accelerometers are commonly used in laptops to detect changes in motion or orientation. In this action, the accelerometer can sense the sudden downward motion or acceleration associated with a free fall of the laptop. Upon detecting the fall, the accelerometer can trigger a protective mechanism such as immediately turning off the hard drive to prevent damage. So, the function of an accelerometer is required in this action.
  3. Detection of the tilt of the smartphone which results in the rotation of the display between portrait and landscape mode: Accelerometers are commonly used in smartphones to detect changes in orientation. When the smartphone is tilted, the accelerometer can detect the change in position and trigger the automatic rotation of the display between portrait and landscape mode. So, the function of an accelerometer is required in this action.

Q. 85) With reference to the role of bio filters in Recirculating Aquaculture System, consider the following statements:

1. Biofilters provide waste treatment by removing uneaten fish feed.

2. Biofilters convert ammonia present in fish waste to nitrate.

3. Biofilters increase phosphorus as nutrient for fish in water.

How many of the statements given above are correct?

(a) Only one

(b) Only two

(c) All three

(d) None

Answer: (b) Only two

  1. Biofilters provide waste treatment by removing uneaten fish feed: This statement is correct. Biofilters play a crucial role in a Recirculating Aquaculture System (RAS) by providing waste treatment. Uneaten fish feed, along with other organic waste, accumulates in the system. Biofilters house beneficial bacteria that break down and remove the uneaten fish feed and other organic matter, helping to maintain water quality.
  2. Biofilters convert ammonia present in fish waste to nitrate: This statement is correct. One of the primary functions of biofilters in RAS is to convert toxic ammonia, which is excreted by fish as waste, into less harmful nitrate through the nitrification process. Beneficial bacteria in the biofilters convert ammonia into nitrite and then further into nitrate, which is less toxic to fish.
  3. Biofilters increase phosphorus as a nutrient for fish in the water: This statement is not correct. Biofilters primarily focus on waste treatment and nitrification processes in RAS, and they do not directly increase phosphorus as a nutrient for fish. The role of biofilters is to remove organic waste and convert ammonia, not to increase phosphorus levels.

Q.86) Consider the following pairs:

Objects in space: Description

1. Cepheids : Giant clouds of dust and gas in space

2. Nebulae : Stars which brighten and dim periodically

3. Pulsars: Neutron stars that are formed when massive stars run out of fuel and collapse

How many of the above pairs are correctly matched?

(a) Only one

(b) Only two

(c) All three

(d) None

Answer: (a) Only one

  1. Cepheids: Giant clouds of dust and gas in space: This description is incorrect. Cepheids are not giant clouds of dust and gas. Instead, they are a type of variable star that exhibits periodic changes in brightness. Cepheids are used as important distance indicators in astronomy.
  2. Nebulae: Stars which brighten and dim periodically: This description is incorrect. Nebulae are not stars that brighten and dim periodically. Nebulae are vast clouds of dust, gas, and plasma in space. They can be regions of star formation or remnants of exploded stars.
  3. Pulsars: Neutron stars that are formed when massive stars run out of fuel and collapse: This description is correct. Pulsars are highly magnetized, rotating neutron stars that emit beams of electromagnetic radiation. They are formed when massive stars undergo supernova explosions and collapse under their own gravitational forces.

Q. 87) Which one of the following countries has its own Satellite Navigation System?

(a) Australia

(b) Canada

(c) Israel

(d) Japan

Answer:  (d) Japan

Among the given options, Japan has its own Satellite Navigation System known as the Quasi-Zenith Satellite System (QZSS). QZSS is a satellite-based positioning system designed to augment and enhance the Global Positioning System (GPS) in Japan and the Asia-Oceania region. It provides improved positioning, navigation, and timing services.

Australia does not have its own satellite navigation system. It relies on the Global Positioning System (GPS) and other satellite navigation systems for positioning and navigation.

Canada also does not have its own satellite navigation system. It primarily relies on the GPS system for positioning and navigation.

Israel does not have its own satellite navigation system. It primarily uses the GPS system for positioning and navigation.

Therefore, the Answer :  is (d) Japan.

Q. 88) Consider the following statements:

1. Ballistic missiles are jet-propelled at subsonic speeds throughout their flights, while cruise missiles are rocket-powered only in the initial phase of flight.

2. Agni-V is a medium-range supersonic cruise missile, while BrahMos is a solid-fuelled intercontinental ballistic missile.

Which of the statements given above is/are correct?

(a) 1 only

(b) 2 only

(c) Both 1 and 2

(d) Neither 1 nor 2

Answer: (d) Neither 1 nor 2

Both statements 1 and 2 are incorrect.

Statement 1: Ballistic missiles are not jet-propelled at subsonic speeds throughout their flights. Ballistic missiles are propelled by rocket engines during their flight trajectory, reaching supersonic or hypersonic speeds. They follow a parabolic trajectory, primarily relying on the initial propulsion to reach their targets.

Statement 2: Agni-V is an intercontinental ballistic missile (ICBM), not a medium-range supersonic cruise missile. BrahMos is a supersonic cruise missile, but it is not solid-fueled and it does not have intercontinental range.

Therefore, the Answer :  is (d) Neither 1 nor 2. Once again, I apologize for the earlier incorrect response.

89. Consider the following statements regarding mercury pollution

1. Gold mining activity is a source of mercury pollution in the world.

2. Coal-based thermal power plants cause mercury pollution.

3. There is no known safe level of exposure to mercury.

How many of the above statements are correct?

(a) Only one

(b) Only two

(c) All three

(d) None

Answer :  is (c) All three

  1. Gold mining activity is a source of mercury pollution in the world: This statement is correct. Gold mining, particularly artisanal and small-scale gold mining (ASGM), is a significant source of mercury pollution globally. Mercury is often used in the gold extraction process, leading to the release of mercury into the environment, which can have adverse effects on ecosystems and human health.
  2. Coal-based thermal power plants cause mercury pollution: This statement is correct. Coal combustion in thermal power plants is a significant contributor to mercury pollution. When coal is burned, mercury present in the coal is released into the atmosphere as mercury emissions. These emissions can then be deposited into water bodies and soil, posing environmental and health risks.
  3. There is no known safe level of exposure to mercury: This statement is correct. Mercury is a toxic heavy metal that can have harmful effects on human health, particularly on the nervous system. Even at low levels of exposure, there is no known safe threshold for mercury. It can accumulate in the body over time and cause various health problems, including neurological disorders and developmental issues, especially in vulnerable populations such as infants and children.

90. With reference to green hydrogen, consider the following statements:

1. It can be used directly as a fuel for internal combustion.

2. It can be blended with natural gas and used as fuel for heat or power generation.

3. It can be used in the hydrogen fuel cell to run vehicles.

How many of the above statements are correct?

(a) Only one

(b) Only two

(c) All three

(d) None

 Answer: (c) All three

  1. Green hydrogen can be used directly as a fuel for internal combustion: This statement is correct. Green hydrogen, which is produced using renewable energy sources, can be utilized as a fuel for internal combustion engines. It can be used as a direct replacement for fossil fuels in vehicles and other applications.
  2. Green hydrogen can be blended with natural gas and used as fuel for heat or power generation: This statement is correct. Green hydrogen can be blended with natural gas in certain proportions and used as a fuel for heat or power generation. This blend, often referred to as “hydrogen-enriched natural gas,” can help reduce the carbon emissions associated with natural gas combustion.
  3. Green hydrogen can be used in the hydrogen fuel cell to run vehicles: This statement is correct. Green hydrogen can be utilized in fuel cells to produce electricity. Hydrogen fuel cells can power vehicles, providing an alternative to conventional internal combustion engines and reducing greenhouse gas emissions.

Q.91)   Consider the following statements with reference to India: 

1. According to the Micro, Small and Medium Enterprises Development (MSMED) Act, 2006′, the ‘medium enterprises’ are those with investments in plant and 15 crore and machinery between 25 crore.

2. All bank loans to the Micro, Small and Medium Enterprises qualify under the priority sector. 

Which of the statements given above is/are correct? 

(a) 1 only 

(b) 2 only 

(c) Both 1 and 2 

(d) Neither 1 nor 2 

Answer:  D

As per the MSME classification criteria, the investment in plant and machinery equipment of Medium enterprises is not more than 20 crore, and the annual turnover is not more than 100 crore. 

Hence statement 1 is wrong.

In terms of Master Direction on ‘Priority Sector Lending (PSL) – Targets and Classification’ dated September 4, 2020, all bank loans to MSMEs conforming to the conditions prescribed therein qualify for classification under priority sector lending.

Hence statement 2 is wrong.

Q.92) With reference to Central Bank digital currencies, consider the following statements:

1. It is possible to make payments in a digital currency without using the US dollar or SWIFT system.

2. A digital currency can be distributed with conditions programmed into it such as a time- frame for spending it.

Which of the statements given above is/are correct?

(a) 1 only

(b) 2 only

(c) Both 1 and 2

(d) Neither 1 nor 2

Answer: (c) Both 1 and 2

  1. It is possible to make payments in a digital currency without using the US dollar or SWIFT system: This statement is correct. Central Bank Digital Currencies (CBDCs) can provide an alternative means of payment that is independent of traditional systems such as the US dollar or SWIFT (Society for Worldwide Interbank Financial Telecommunication). CBDCs are digital representations of a country’s fiat currency issued and regulated by the central bank. Payments can be made directly in CBDCs, reducing reliance on intermediaries and potentially bypassing systems like the US dollar or SWIFT.
  2. A digital currency can be distributed with a condition programmed into it, such as a time-frame for spending it: This statement is also correct. CBDCs can be programmable currencies, allowing conditions to be attached to their use. For example, a central bank could program a CBDC with an expiration date or time frame for spending it, encouraging prompt circulation of the currency and stimulating economic activity.

Q93. In the context of finance, the term “beta” refers to

(a) the process of simultaneous buying and selling of an asset from different platforms

(b) an investment strategy of a portfolio manager to balance risk versus reward

(c) a type of systemic risk that arises where perfect hedging is not possible

(d) a numeric value that measures the fluctuations of a stock to changes in the overall stock market

Answer: (d) a numeric value that measures the fluctuations of a stock to changes in the overall stock market.

Beta is a measure of the sensitivity or volatility of a stock or investment portfolio in relation to the broader market. It quantifies the extent to which the price of a particular stock tends to move in relation to movements in a benchmark index, such as the overall stock market.

A beta value greater than 1 indicates that the stock is more volatile than the market, meaning it tends to have larger price swings. A beta value less than 1 suggests that the stock is less volatile than the market, indicating relatively smaller price movements. A beta of 1 means the stock generally moves in line with the market.

Investors and analysts use beta as a tool to assess the risk of a stock or portfolio and understand how it may behave in different market conditions. It helps investors determine the potential volatility and performance of an investment relative to the overall market.

Therefore, the Answer   (d) a numeric value that measures the fluctuations of a stock to changes in the overall stock market.

Q94. Consider the following statements:

1. The Self-Help Group (SHG) programme was originally initiated by the State Bank of India by providing microcredit to the financially deprived.

2. In an SHG, all members of a group take responsibility for a loan that an individual member takes.

3. The Regional Rural Banks and Scheduled Commercial Banks support SHGs.

How many of the above statements are correct?

(a) Only one

(b) Only two

(c) All three

(d) None


Answer: (b) Only two

  1. The Self-Help Group (SHG) programme was originally initiated by the State Bank of India by providing microcredit to the financially deprived: This statement is incorrect. The Self-Help Group (SHG) movement in India was not originally initiated by the State Bank of India. It was started by non-governmental organizations (NGOs) and other grassroots organizations to address the financial needs of the marginalized and financially deprived sections of society.
  2. In an SHG, all members of a group take responsibility for a loan that an individual member takes: This statement is correct. In a typical SHG model, all members collectively take responsibility for the loans taken by individual members of the group. Members contribute to a common fund, and loans are given to members from this pooled fund. The group members jointly ensure the repayment of the loans taken by any individual member.
  3. The Regional Rural Banks and Scheduled Commercial Banks support SHGs: This statement is correct. Regional Rural Banks (RRBs) and Scheduled Commercial Banks (SCBs) actively support SHGs in India. They provide financial assistance, including credit and banking services, to SHGs. These banks often collaborate with NGOs and government agencies to promote the SHG movement and provide financial support to the members.

Q95. Consider the following statements:

Statement-I: India’s public sector health care system largely focuses on curative care with limited preventive, promotive and rehabilitative care.

Statement-II: Under India’s decentralized approach to health care delivery, the States are primarily responsible for organizing health services.

Which one of the following is correct in respect of the above statements?

(a) Both Statement-I and Statement-II are correct and Statement-II is the correct explanation for Statement-I

(b) Both Statement-I and Statement-II are correct and Statement-II is not the correct explanation for Statement-I

(c) Statement-l is correct but Statement-II is incorrect

(d) Statement-I is incorrect but Statement-II is correct

Answer: (b) Both Statement-I and Statement-II are correct, but Statement-II is not the correct explanation for Statement-I

Statement-I: India’s public sector health care system largely focuses on curative care with limited preventive, promotive, and rehabilitative care. This statement is correct. India’s public sector health care system has historically been more focused on curative care, with a significant emphasis on hospitals and specialized medical treatment. However, in recent years, there has been a growing recognition of the need to strengthen preventive, promotive, and rehabilitative care to improve overall health outcomes.

Statement-II: Under India’s decentralized approach to health care delivery, the States are primarily responsible for organizing health services. This statement is also correct. India follows a decentralized approach to health care delivery, where the responsibility for organizing and implementing health services lies primarily with the state governments. The central government provides guidance, policy frameworks, and financial support to the states, but the states have the autonomy and responsibility to plan, manage, and deliver health care services based on local needs and priorities.

While both statements are correct, Statement-II is not the correct explanation for Statement-I. The focus of India’s public sector health care system on curative care is not solely due to the decentralized approach to health care delivery. It is a broader aspect related to the historical development and priorities of the health care system.

Therefore, the Answer :  (b) Both Statement-I and Statement-II are correct, but Statement-II is not the correct explanation for Statement-I.

96. Consider the following statements:

Statement-I: According to the United Nations World Water Development Report, 2022′, India extracts more than a quarter of the world’s groundwater withdrawal each year.

Statement-II: India needs to extract more than a quarter of the world’s groundwater each year to satisfy the drinking water and sanitation needs of almost 18% of world’s population living in its territory.

Which one of the following is correct in respect of the above statements?

(a) Both Statement-I and Statement-II are correct and Statement-II is the correct explanation for Statement-l

(b) Both Statement-I and Statement-II are correct and Statement-II is not the correct explanation for Statement-I

(c) Statement-I is correct but Statement-II is incorrect

(d) Statement-I is incorrect but Statement-II is correct

Answer: (c) Statement-I is correct but Statement-II is incorrect

Statement-I: According to the United Nations World Water Development Report, 2022, India extracts more than a quarter of the world’s groundwater withdrawal each year. This statement is correct. The report highlights that India is one of the countries with significant groundwater extraction, accounting for more than 25% of the global groundwater withdrawal annually.

Statement-II: India needs to extract more than a quarter of the world’s groundwater each year to satisfy the drinking water and sanitation needs of almost 18% of the world’s population living in its territory. This statement is incorrect. While India does have a large population and faces significant challenges in meeting its water needs, it does not need to extract more than a quarter of the world’s groundwater to fulfill drinking water and sanitation requirements. The water needs of a population are influenced by various factors, including water availability, efficiency, management, and infrastructure. 

97. Consider the following statements:

1. According to the Constitution of India, the Central Government has a duty to protect States from internal disturbances.

2. The Constitution of India exempts the States from providing legal counsel to a person being held for preventive detention.

3. According to the Prevention of Terrorism Act, 2002, confession of the accused before the police cannot be used as evidence.

How many of the above statements are correct?

(a) Only one

(b) Only two

(c) All three

(d) None

Answer: (a) Only one.

Statement 1 is correct: According to the Constitution of India, the Central Government has a duty to protect States from internal disturbances. This is mentioned in Article 355 of the Constitution, which states that it is the duty of the Union to protect every State against external aggression and internal disturbance.

 Statement 2 is incorrect: The Constitution of India does not exempt the States from providing legal counsel to a person being held for preventive detention. The right to legal counsel is a fundamental right under Article 22 of the Constitution, which applies to all persons, including those in preventive detention.

Statement 3 is incorrect: The Prevention of Terrorism Act, 2002 has been repealed. It was replaced by the Unlawful Activities (Prevention) Act, 1967. The admissibility of confession as evidence is determined by the provisions of the Indian Evidence Act, and it is not categorically prohibited solely based on the law related to terrorism.

98. Which one of the following countries has been suffering from decades of civil strife and food shortages and was in news in the recent past for its very severe famine?

(a) Angola

(b) Costa Rica

(c) Ecuador

(d) Somalia

Answer: (d) Somalia

Somalia has been suffering from decades of civil strife, political instability, and recurring conflicts. It has also faced severe food shortages and humanitarian crises, including famines. In the recent past, Somalia has been in the news for experiencing very severe famines and humanitarian emergencies, with millions of people facing food insecurity and malnutrition. The country has been grappling with complex challenges related to governance, security, and access to basic services, leading to a persistent humanitarian crisis.

99. Consider the following statements:

1. In India, the Biodiversity Management Committees are key to the realization of the objectives of the Nagoya Protocol.

2. The Biodiversity Management Committees have important functions in determining access and benefit sharing, including the power to levy collection fees on the access of biological resources within its jurisdiction.

Which of the statements given above is/are correct?

(a) 1 only

(b) 2 only

(c) Both 1 and 2

(d) Neither 1 nor 2

Answer: (c) Both 1 and 2

  1. In India, the Biodiversity Management Committees are key to the realization of the objectives of the Nagoya Protocol: This statement is correct. The Nagoya Protocol is an international agreement under the Convention on Biological Diversity (CBD), which aims to ensure the fair and equitable sharing of benefits arising from the utilization of genetic resources. In India, the Biodiversity Management Committees (BMCs) play a crucial role in implementing the Nagoya Protocol at the local level. They are responsible for promoting conservation, sustainable use, and equitable sharing of benefits derived from biological resources within their jurisdiction.
  2. The Biodiversity Management Committees have important functions in determining access and benefit sharing, including the power to levy collection fees on the access of biological resources within its jurisdiction: This statement is also correct. The Biodiversity Management Committees (BMCs) in India have several functions related to access and benefit sharing (ABS) of biological resources. They have the authority to grant permissions for access to biological resources, ensure compliance with ABS provisions, and determine the sharing of benefits arising from the utilization of these resources. The BMCs can also levy collection fees or charges on the access of biological resources within their jurisdiction as a means of generating funds for biodiversity conservation and benefit sharing.

100. Consider the following statements in respect of election to the President of India:

1. The members nominated to either House of the Parliament or the Legislative Assemblies of States are also eligible to be included in the Electoral College.

2. Higher the number of elective Assembly seats, higher is the value of vote of each MLA of that State.

3. The value of vote of each MLA of Madhya Pradesh is greater than that of Kerala.

4. The value of vote of each MLA of Puducherry is higher than that of Arunachal Pradesh because the ratio of total population to total number of elective seats in Puducherry is greater as compared to Arunachal Pradesh.

How many of the above statements are correct?

(a) Only one

(b) Only two

(c) Only three

(d) All four

Answer: (a) Only One

Statement 4 is correct: The value of the vote of each Member of Legislative Assembly (MLA) in the Presidential election is determined based on the ratio of the population of the state to the number of elected seats in the legislative assembly. Therefore, the value of the vote of each MLA can vary from state to state based on this calculation.

Statements 1, 2, and 3 are incorrect:

  • Statement 1: The members nominated to either House of Parliament or the Legislative Assemblies of States are not included in the Electoral College for the election of the President of India. Only elected members of both Houses of Parliament and the Legislative Assemblies of States are part of the Electoral College.
  • Statement 2: The value of the vote of each MLA is not directly proportional to the number of elective assembly seats. It is determined based on the population of the state and the number of elected seats in the assembly.
  • Statement 3: The value of the vote of each MLA is not directly related to the population of the state. It is determined based on the formula mentioned above.

Therefore, the correct answer is (d) All four.


Subscribe
Notify of
0 Comments
Inline Feedbacks
View all comments

JOIN THE COMMUNITY

Join us across Social Media platforms.

💥Mentorship New Batch Launch
💥Mentorship New Batch Launch